OJ EPO SE 3/2018, p2 - BOARDS OF APPEAL AND ENLARGED BOARD OF APPEAL CASE LAW 2017

I. PATENTABILITY

A. Exceptions to patentability

1. Essentially biological processes for the production of plants

(Case Law of the Boards of Appeal, 8th ed. 2016 ("CLB"), I.B.3.3.2)

In T 2323/11 the invention concerned the removal of unwanted sequences from the genome of transgenic plants which comprised an introduced gene for an agronomically valuable trait. The claimed method concerned the sexual crossing of two distinct transgenic parent plants. The first transgenic parent plant contained, besides sequences of interest conferring a particular trait on the B. plant, an undesired gene sequence in its genome ("target gene"). The second transgenic parent plant comprised a transgene encoding a particular sequence-specific DNA endonuclease which, when expressed in a cell which comprised the "target gene" in its genome, irreversibly excised this target gene from the genome. From such crosses, descending plants could be isolated which lacked the "target gene" sequences in their genome. Claim 1 was directed to those steps in the production process of a transgenic plant which involved the crossing of two particular plants and the generation of first generation descendants thereof and the isolation and selection of those descendants which had lost the unwanted sequences in their genome. Accordingly, the board considered that Art. 53(b) EPC and decisions G 2/07 (OJ 2012, 130) and G 1/08 (OJ 2012, 206) were applicable. In G 2/07 and G 1/08 the Enlarged Board considered that, under Art. 53(b) EPC, excluded processes were characterised by the fact that the traits of the plants resulting from the crossing were determined by the underlying natural phenomenon of meiosis. This phenomenon determined the genetic make-up of the plants produced, and the breeding result was achieved by the breeder's selection of plants having the desired traits. The appellant argued that, in the claimed method, the step of the excision of the target gene was not the result of meiosis and the mixing of the parental genes, but that this process step took place at mitotic stages after the mixing of the genes and the formation of the zygote. The appellant therefore considered that the claimed method fell under the exception made in G 2/07 and G 1/08, namely: "[i]f, however, such a process contains within the steps of sexually crossing and selecting an additional step of a technical nature, which step by itself introduces a trait into the genome or modifies a trait in the genome of the plant produced, so that the introduction or modification of that trait is not the result of the mixing of the genes of the plants chosen for sexual crossing, then the process is not excluded from patentability under Art. 53(b) EPC". The board stated that while the factual excision of the target gene occurred after the zygote had been formed, the presence (or absence) of the features in the genomes of the respective gametes which fused to generate the desired zygote, and thus the plant resulting from the crossing, was determined by meiosis. It therefore considered that the trait of the excision of the target gene was the result of the crossing of the parent plants and was determined by the underlying natural phenomenon of meiosis, as the latter determines the genetic make-up of the plants produced. The claimed method was thus excluded from patentability.

2. Therapeutic methods

2.1 Methods with both therapeutic and non-therapeutic indications

(CLB, I.B.4.4.2)

In T 158/13 the claims were directed to the non-therapeutic use of a stimulative perfume composition "for releasing persons from a physiological mental condition of sleepiness, sense of fatigue and inactivity in daily life, and for refreshing and activating their mental condition". The opposition division found that the subject-matter met the requirements of Art. 53(c) EPC, since it was possible to clearly distinguish between a non-therapeutic and a therapeutic use of the stimulative perfume in question because it was possible to distinguish between persons that only had little sleep and those who suffered from chronic fatigue or depression, and consequently the non-therapeutic use arose from a proper selection of persons on which the stimulative perfume was used. However, the appellant pointed out that it was not possible to distinguish between a therapeutic or non-therapeutic use of the stimulative perfume composition, since the way of administering the composition was the same for a therapeutic and a non- therapeutic use. Moreover, the claimed use covered at least some mental conditions, such as sleepiness, that might or might not be of a pathological nature.

According to the case law of the boards of appeal a clear distinction between a therapeutic and a non-therapeutic use was possible if the group of persons treated could be clearly identified as healthy or as suffering from a pathological condition (T 469/94). In the present case at least, the sleepiness or inactivity in daily life might not only be a physiological condition but might also arise from a pathological condition. Without a clear and unambiguous distinction between a physiological and a pathological nature of a person's mental condition, a clear distinction between therapeutic and non-therapeutic use was also impossible. The mere reference to the claimed use being a "non-therapeutic" use did not, therefore, exclude the therapeutic use. Since for the use for some of the claimed mental conditions a clear distinction between therapeutic and non-therapeutic use could not be made the requirements of Art. 53(c) EPC were not fulfilled.

In T 44/12 the appeal of the patent proprietor was against the decision of the opposition division to revoke the patent as a method for treatment of the human body by therapy pursuant to Art. 53(c) EPC. The opposition division and the respondents (opponents) considered that the step "initiating the pumping sequence to cause the fluid to flow into the fluid line" of claim 2 was a therapeutic step because it changed the patient's status through the injection of medicine. Claim 1, for "a method of automatically detecting an occlusion in a downstream fluid line of a medical pumping system", was also deemed to fall under Art. 53(c) EPC, because it encompassed the surgical step of placing the infusion needle and the therapeutic step of pumping medicine into the patient during the execution of the claimed method. The board disagreed. According to the board, whether or not a method was to be considered a method for treatment of the human body by therapy was determined by the existence of a functional link or a direct influence of the method claimed in itself on a given therapy, such that the medical doctor's freedom of choice and of practice in this respect was hindered. The board considered that this was not the case for the present method claims. The necessary puncturing of the patient to connect him to the pumping system was irrelevant for the question of whether or not the method falls under the exceptions to patentability pursuant to Art. 53(c) EPC. Hence, in the present case the board considered that the subject-matter of claims 1 and 2 only concerned the operation of a device, without any functional link to the effects of the device on the body, and so the method did not fall under the exception to patentability pursuant to Art. 53(c) EPC concerning methods for treatment of the human body by therapy or by surgery.

B. Novelty

1. State of the art

(CLB, I.C.2)

In ex parte case T 1090/12, the appellant drew attention to the Guidelines G-VII, 3.1, which reads: "an assertion that something is common general knowledge need only be backed by documentary evidence (for example, a textbook) if this is contested", and submitted that this passage was binding on the boards of appeal when exercising the power of the examining division. In the case in hand, the board denied that it was obliged to provide written evidence and explained during the oral proceedings that its members knew from their work on previous cases that the features at issue were known to the person skilled in the art. It would therefore have been for the appellant to show an error in the board's understanding, e.g. that one of the features at issue was in fact introduced into the art after the priority date of the application under appeal. There was also no general obligation on a board to provide documentary evidence of the existence of a piece of common general knowledge. In proceedings before the EPO, a board of appeal had to respect the right to be heard and to give reasons for its decision. In cases where a board referred to the common general knowledge as state of the art, it was not obliged to provide documents in every conceivable case. It was also possible for a board to state what it deemed to be known, and possibly where it was known from, in a way that would put the appellant in a position to try to convince the board that its findings were erroneous. Proceeding in this way respected the party's right to be heard. The board also recalled that there was no obligation on the part of a board of appeal to follow the Guidelines. In this respect, the request for referral of questions to the Enlarged Board of Appeal was refused.

2. Availability to the public

2.1 Advertising brochure

(CLB, I.C.3.2.1 c))

In T 523/14, concerning the public prior availability of D11 which, was the fall 2007 issue of the advertising newsletter "Glasstech World" of respondent 1 (opponent 1), it was disputed among the parties whether it had been made available to the public before the filing date of the patent (26 February 2008). Practically all the evidence in support of the alleged prior publication of D11 lay within the power and knowledge of respondent 1, with the appellant having no ready access to it. For such a situation, it is established case law that the allegations must be proven "beyond all reasonable doubt", namely that the allegations need to be proven in such a manner that the board, on the basis of a free evaluation of the evidence on file, is satisfied that the alleged facts have actually occurred. After reviewing the evidence on file, the board was satisfied that D11 had indeed been made available.

Considering the enabling disclosure in D11, the board stated that it was common ground that D11, being an advertising newsletter, was not drafted in a manner as rigorous as for a scientific publication. Nevertheless, the information given in D11 was sufficient to enable a skilled reader, at the relevant date of D11, to practise the technical teaching which was the subject of the document, taking into account also common general knowledge at that time in the field of glass-processing. The present case was not comparable to T 412/91.

2.2 Public prior use

2.2.1 Internal structure or composition of a product

(CLB, I.C.3.2.4 d))

In T 2048/12 the invention concerned a chemical compound TMAEE as a catalyst for polyurethane. The appellant (opponent) raised an objection of prior use, namely that TMAEE was present as an impurity in a chemical product already commercially available. The board stated that opinion G 1/92 did not imply that in each and every case the commercial availability of a chemical product as such necessarily amounted to a disclosure of (also) all the impurities contained therein merely because it is possible to identify and quantify these impurities by analytical means. Conclusion 1 of G 1/92 was to be read by attributing a technically reasonable meaning to the technical expression "chemical composition". In the case in issue there was no direct or indirect pointer to the possible technical relevance of further impurities (besides water) in this commercial product.

2.3 Obligation to maintain secrecy

(CLB, I.C.3.4)

In T 1829/12, the opposition division had concluded that the opponent had failed to prove that a sensor unit allegedly in prior use had been publicly available, and therefore held that the unit could not be regarded as prior art for the purposes of Art. 54(2) EPC. It had found that supplier R had concluded an exclusive delivery contract including a non-disclosure clause with company I and similarly agreed a confidentiality obligation with company T. In both cases, therefore, its deliveries of the sensor unit or device had been covered by a non-disclosure agreement.

On appeal, it appeared not be in dispute that R and T had expressly entered into such an agreement, but the appellant (opponent) contended that it had been of limited scope and had not covered the sensor unit. The board rejected that contention because it ran entirely counter to normal business practice for a non-disclosure agreement not to cover the very technical means needed to achieve the intended aim.

According to the parties' submissions, R and I had not documented any non-disclosure agreement in writing but had merely entered into an "exclusive delivery contract", so the question here was whether an implied confidentiality obligation could arise from, for instance, special business or contractual relations or joint development activities. The board found that the two companies' business and contractual relations under the exclusive contract supported the view that they were subject to an implied confidentiality obligation and endorsed the opposition division's findings accordingly.

2.4 Making samples/products available for test purposes

(CLB, I.C.3.4.7)

In T 2068/15 the opposition division decided that it had been proven beyond reasonable doubt by written evidence and by the testimony of a witness that 37 films had been sold and delivered to Nissan without any obligation of secrecy. These films had therefore been made public. However, it had not been proven up to the hilt that these films had first and second layers as claimed.

The board accepted that, if joint development projects were agreed, they were often – explicitly or implicitly – combined with an obligation of confidentiality. In the case in hand, however, the board did not see enough indications from which it could be convincingly concluded that there existed a joint development project between opponent 2 and Nissan. The circumstances were suggestive of ordinary sales rather than a sending of samples within the framework of a joint development project. The board concluded that the films themselves were not to be treated as confidential.

The proprietor argued that even if the 37 films sold had been public, they would still not have been novelty-destroying, since at the priority date of the patent the skilled person would not have been able to gain knowledge about the chemical composition of the films sold. In particular, if the three research institutes that carried out the tests had not been informed beforehand that the film was composed of three layers, they would have overlooked the very thin first top layer. The proprietor further argued that they would not have been able to find the relevant reference spectra needed.

The board acknowledged that a precondition for the chemical composition of a product to be prior art is that it can be analysed by the skilled person (G 1/92, point 1 of the headnote, OJ 1993, 277). But the board considered in the case at issue that the skilled person analysing the film with the techniques known at that time (electron microscopy) would not have overlooked the first top layer. And the board also stated that it was common practice at the priority date of the patent to apply more than one analytical method in order to obtain information about the composition of a material. In addition, even if, as argued by the proprietor, the IR reference spectrum for the polycaprolactone-based polyurethane of the second layer was not available at the priority date, the skilled person could still have determined the composition of this layer.

3. Determining the content of the relevant prior art

3.1 General rules of interpretation

(CLB, I.C.4.1)

In T 1658/12 the board stated that in determining what was made available to the public within the meaning of Art. 54(2) EPC by a prior-art patent document, it had to be borne in mind that it was the description which chiefly served to disclose the invention in such a manner that it might be carried out, whereas the chief function of the claims was to define the subject-matter for which protection was sought. Where a combination of features is found only in the claims (or only in the claims and a "Summary of the Invention", which merely recites the features of the claims), it must be very carefully considered whether this combination truly corresponds to the technical teaching of the document as it would be understood by a skilled person, or whether it is merely an artefact of the claim drafting process aimed at obtaining the maximum scope of protection (see point 3.8 of the Reasons; also T 312/94, catchword; T 969/92, point 3 of the Reasons, and in particular, page 4, first paragraph; T 42/92). This applied all the more in the case of unexamined applications.

4. First and second medical use

4.1 Interpretation of "substance or composition"

(CLB, I.C.7.2.4 g))

In T 1758/15 the patent related to a biocompatible, biodegradable, injectable filler material (e.g. collagen or hyaluronic acid) for use in a specific method. Said use comprised the injection of the filler material into a space between a first tissue of the body and a second tissue, the first tissue being subsequently treated by radiation, whereby the filler within the space reduced the exposure of the second tissue to radiation. The filler "human collagen" was known in the art. Hence, the board had to examine whether the filler material claimed fell under the term "substance or composition" required by Art. 54(5) EPC for purpose-limited product protection.

In the interpretation of the term "substance or composition" the board referred to G 5/83 (OJ 1985, 64) and T 2003/08, which interpreted the term "substance or composition" as being "the active agent or ingredient" of the particular specific medical use. Accordingly, the following must be established: (a) the means by which the therapeutic effect is achieved and (b) whether that which achieves the therapeutic effect is a chemical entity or composition of chemical entities. In the present case, the effect of the use of the filler material was the reduction of radiation-treatment-induced side effects on sensitive organs. This effect was constantly ascribed to the physical displacement of the sensitive tissue, i.e. to the 3D structure achieved within the body. The accumulated mass of the filler material did not, however, qualify as a chemical entity or composition of chemical entities in the sense of G 5/83.

Therefore, the filler material was not a substance or composition in the sense of Art. 54(5) EPC and its specific use could not be regarded as a differentiating feature. The undisputedly commercially available collagen was thus novelty-destroying.

C. Inventive step

1. Closest prior art

1.1 Speculative character

(CLB, I.D.3.4.6)

In T 725/11 the invention was directed to a pharmaceutical co-formulation in the form of a tablet comprising two active ingredients for HIV therapy. The board did not acknowledge an inventive step over an announcement by the patentee of a clinical trial of that combination therapy in an industry journal article. The patentee argued that this journal article was not the closest prior art because it was silent on efficacy and did not provide any technical details. The board disagreed and stated that the journal article amounted to a concrete plan to develop a commercially viable product with a usable level of efficacy. Furthermore, the article was a public statement of intent made by the patentee's CEO and its executive vice president of research and development which would not be dismissed by the skilled person as mere speculation.

2. Assessment of inventive step

2.1 Technical character of the invention

(CLB, I.D.9.1.1)

In T 630/11 the invention was about online gambling. It provided a "gaming server" with which various online casinos communicate. The description explained that the principal aim of the invention was to reduce the time that players spend waiting. The invention involved technical and non-technical considerations. According to the board, it is well established that non-technical elements do not contribute to inventive step. That was the basic principle of T 641/00, OJ 2003, 352 (Comvik). Comvik set out an approach to ensuring that non-technical issues do not influence the decision on inventive step. This was to include the non-technical elements in the statement of the technical problem to which the skilled person seeks a solution. Most often, this was in the form of non-technical requirements. In T 1463/11 the board had stated that the notional business person (more generally, the non-technical person) cannot normally require even notorious technical means. The reason was that the inventor may have obtained a technical effect using technical means, even notorious means, in a way that would not have been obvious to the skilled person. That was what a patent was meant to reward. To allow the notional business person to prescribe technical means would be to foreclose any discussion of whether they were used in a technically non-obvious way. This should not lead to a proliferation of patents for technically trivial inventions; they would be obvious to the skilled person.

2.2 Assessment of features relating to mathematical algorithms

(CLB, I.D.9.1.8)

At issue in T 625/11 was a method to establish at least one threshold value of a parameter for operating a nuclear reactor with a view to making better use of the reactor's capacity. The appeal was against the examining division's decision to refuse the European patent application for lack of inventive step. The examining division had taken the view that the various method steps had no technical character.

The board observed that a technical problem could be defined only if at least one technical effect was achieved by at least one of the claimed features (see T 641/00). In this case, the first claim's wording left no doubt that the steps in the claimed method were performed by the computer system it referred to. The board thus found that not only the claim as a whole but, given this need to use the computer system, each of the method steps too were technical.

In this case, however, that alone was not enough to establish whether or not the claimed method was inventive for the purposes of Art. 56 EPC 1973. The appellant had argued that the method's technical character resulted not only from the use of a computer system but also from the data processed and the invention's intended purpose, even if it did not form part of the claimed method.

The board found that establishing a threshold value for the first operating parameter lent the claim a technical dimension going beyond the mere interaction of the numerical-simulation algorithm and the computer system, the established parameter having a purpose closely linked to operating a nuclear reactor, regardless of whether or not it was actually used for that. The board thus acknowledged that the approach taken in T 1227/05 (OJ 2007, 574) was appropriate, and applied it to the circumstances of this case. The simulation parameters (stress, temperatures, heat capacity, pressure, dimensions, etc.) were similarly of a nature to give the claimed invention technical character. On this basis, the board concluded that the first claim of the main request was inventive.

2.3 Example of lack of inventive step

(CLB, I.D.9.18)

In T 519/12 the board held that it had to be expected of the skilled person that he would exercise his skills in the framework of technical standards in force in his field of activity. No inventive activity could be derived from a feature that simply reflected the contents of such a technical prescription.

3. Secondary indicia in the assessment of inventive step

(CLB, I.D.10)

In T 1892/12, the board observed that the boards had in some cases taken account of "secondary indicia" that an inventive step was involved but had not so far done so for the purposes of establishing that it was lacking. According to Art. 56 EPC, an invention shall be considered as involving an inventive step if, having regard to the state of the art, it is not obvious to a person skilled in the art. In view of the negative definition in Art. 56 EPC, the board considered that inventive step could never in fact be proven but that secondary indicia might sometimes show it was plausible. A lack of inventive step by contrast could actually be proven by conclusively showing that a solution was obvious. So proving a lack of inventive step was possible, in principle, and indeed necessary where such a lack was alleged; it was then not enough to point to secondary indicia.

II. PATENT APPLICATION AND AMENDMENTS

A. Claims

1. Clarity of claims

(CLB, II.A.3)

In T 268/13 the board concluded that a claim to a "method for producing a [...] decorative strip having a [...] structure consisting of a text or graphic symbol [...]" satisfied the requirements of Art. 84 EPC 1973. Although it was impossible to give an entirely general, universally applicable definition of what constituted a symbol, in the case in hand it would be clear whether or not a particular character was a symbol in the given cultural, linguistic or technical context. The claim therefore presented neither somebody wanting to avoid infringing the patent nor a court deciding on infringement with an impossible task.

2. Interpretation of claims

(CLB, II.A.6)

In T 1513/12 the board noted that an interpretation of a claim agreed by the parties to the proceedings was not to be regarded as binding on the board: the "principle of party disposition" was not to be understood as meaning that the parties to the proceedings could choose an interpretation of the patent which, although it might be satisfactory for them, might have implications for others not party to the proceedings.

B. Unity of invention

1. Unsearched subject-matter and applicability of Rule 137(5) EPC or former Rule 164(2) EPC

(CLB, II.B.6.3)

For the Euro-PCT application in case T 145/13 the examining division did not admit the first auxiliary request (now the main request) under R. 137(5) EPC because certain features of claim 1 of this request had formerly constituted the fifth and unsearched invention identified in the supplementary European search report. However, the appellant argued that each of these features related to a further limitation of the searched automotive equipment control system.

The board noted that the first sentence of R. 137(5) EPC (version in force from 1 April 2010, see OJ 2009, 299) was identical to R. 86(4) EPC 1973, which was introduced to avoid an applicant switching, e.g. by incorporating a feature from the description, to unsearched subject-matter which was not claimed at the search stage (T 442/11). However, the subject-matter of claim 1 corresponded essentially to a combination of the features of claims underlying the supplementary search report. Therefore the claims could not be considered as having been amended such that they related to unsearched subject-matter which did not combine with parts of the originally claimed invention to form a single general inventive concept (T 507/11, T 1285/11, T 1981/12 and T 998/14). R. 137(5) EPC was therefore not contravened.

The board also considered (former) R. 164(2) EPC (OJ 2009, 582). Its second alternative was intended to prevent switching during the grant proceedings from searched subject-matter to subject-matter originally claimed but not searched due to non-payment of an additional search fee (T 442/11). The contested decision had given no justification for the fifth invention not having been searched. According to the supplementary search report, the claims were considered to lack unity a posteriori in view of D1, which was considered to disclose the subject-matter of claim 1. Claims 30-35, 42 and 45 underlying the supplementary search report had been attributed to the fifth invention with the special technical feature of a device type to be controlled by the automatic vehicle control system, which was considered to solve the objective technical problem of providing an "automatic vehicle control system which is able to control a device of a special type". D1 was considered to disclose an automatic vehicle equipment control system.

According to the board, however, a statement contained in D1 was merely an outlook on various possible future developments, with no enabling disclosure concerning them. D1 was therefore not considered to disclose an automatic vehicle equipment control system. This feature could therefore be considered an inventive concept joining the first and fifth inventions as subdivided in the supplementary search report, and so it was not justified that the fifth invention indicated in the report had not been searched. Consequently, R. 164(2) EPC could not be invoked against the amendments in relation to the claims of the main request either (T 442/11).

C. Sufficiency of disclosure

1. The relationship between Article 83 and Article 84 EPC

(CLB, II.C.7)

In T 2290/12, the board summarised the case law on sufficient disclosure versus clarity (see also similar decisions T 1811/13 in English and T 647/15 in French). The respondent had asserted that the invention at issue could not be carried out (Art. 100(b) EPC 1973), arguing that the skilled person had to know when they were working within the forbidden area of the claims and citing T 464/05 and T 256/87 in support of this position.

The board in T 256/87, having found a claim about a chemical composition to be clear for the purposes of Art. 84 EPC 1973, had gone on to examine whether the technical teaching had been disclosed sufficiently to enable the skilled person to carry out the invention in the sense of being able both to establish whether a composition fell within the claimed range and to make that composition. The T 256/87 approach had thus tied an invention's reproducibility to its precise definition. Board 3.2.06 had followed it in four decisions (T 387/01, T 252/02, T 611/02 and T 464/05), finding that the skilled person had to know whether they were working within the claimed scope of protection in order to be able to carry out the invention.

The board observed, however, that this approach appeared to have been dropped since then. In more than 20 decisions, in particular in the field of chemistry, boards had called it into question and made clear that how the scope of protection was defined was more about clarity (Art. 84 EPC) than about whether the invention was sufficiently disclosed.

The board thus concluded that a broad consensus, or at least a prevailing view, had now been reached that the skilled person's ability to establish whether or not subject-matter fell within the claimed scope was a requirement for clarity and not for sufficient disclosure. The board shared that view.

T 608/07 had moreover highlighted the need to ensure that an objection based on insufficient disclosure was not in fact a hidden clarity objection. That was not to say that a lack of clarity could not result in insufficient disclosure of the invention, but it was not enough in such a case merely to show that the claim was unclear. Rather, it had to be shown that the patent specification as a whole was so unclear that the skilled person – who could also draw on the description and common general knowledge – would be unable to carry out the invention (see T 1886/06, T 593/09).

All the respondent's objections under Art. 100(b) EPC 1973 were concerned with the precise scope of protection conferred and so, in fact, clarity objections. Since the claims in the appellant's main request were those already granted, the board could not consider these objections to their clarity (see G 3/14, OJ 2015, A102).

D. Priority

1. Priority right of the applicant or his successor in title

(CLB, II.D.2.2)

In T 1201/14 the board looked into the question of the formal requirements for the transfer of the right of priority. As the EPC contained neither guidance nor conflict-of-laws provisions for that purpose, national law was commonly relied upon by the departments of the EPO entrusted with the procedure. The board acknowledged that there was no established jurisprudence of the boards as to the national law generally applicable to this question, observing also that the formal requirements of different national laws for the transfer of intellectual property rights ranged from none at all (as e.g. under German law; see German Civil Code BGB, § 398 and § 413) to strict formal rules (as e.g. under US law; see 35 U.S.C. § 261). The board's conclusion was that it had to be proven that the right of priority derived from the first application and claimed for the later European application had indeed been transferred, before the filing date of the later European application, by the applicant of the first application to its successor in title, in accordance with specific formal requirements.

Turning then specifically to the matter of a retroactive transfer such as the nunc pro tunc assignment under US law invoked by the appellant, the board held that even if this was allowable under US law, it would not be acceptable under Art. 87(1) EPC 1973. Any natural or legal person could only be considered a "successor in title" within the meaning of Art. 87(1) EPC 1973 if it obtained that right from its previous owner by a transfer agreement concluded before the filing of the later European patent application.

On the rules applicable for assessing the evidence, the board held that an implied transfer of a particular right could be accepted when it was sufficiently clear that the parties had formed an agreement and what they had agreed. The burden of proving a valid transfer of the right of priority lay with the proprietor since it was the one claiming that right. As to the standard of proof to be applied to an implied transfer by virtue of a general policy under German law, the board held that the circumstances of the case in hand required proof "beyond reasonable doubt", as all the relevant evidence lay within the knowledge and power of only one party to the inter partes proceedings.

2. Disclosure in the previous application as a whole – solution of the same problem

(CLB, II.D.3.1.3, II.D.3.1.6)

In case T 1434/13 the contested patent was for a container for carrying an information storage medium. The opposition division had found that neither P1 nor P2, from which priority was claimed, disclosed certain features of claim 1 and that neither was concerned with the problem solved by the patent, which was displaying additional media format information on the container.

The board applied the strict interpretation of the "same invention" set out in G 2/98 (OJ 2001, 413). The criterion "directly and unambiguously derivable, using common knowledge" was the same as that used for judging whether an amendment fulfilled the requirements of Art. 123(2) EPC.

The board then referred to T 169/83 (OJ 1985, 193), which discussed the restriction of granted claims with features taken from drawings. Conclusions of that decision on the interpretation of features shown solely in drawings had since been considered relevant in different contexts, including those of the priority right and added subject-matter. According to T 169/83, features clearly disclosed only in the original drawings could be used to define more precisely the subject-matter for which protection was sought as long as the skilled person could, on the basis of the whole description, unmistakably and fully recognise those features in terms of their structure and function as being manifestly part of the invention. An amendment was unallowable if the overall change in content of the application, whether by way of addition, alteration or excision, resulted in the skilled person being presented with information not directly and unambiguously derivable from that previously presented by the application. Amendments directed to an undisclosed arbitrary combination of features of different embodiments were not allowable.

According to the board, those findings of T 169/83 equally applied to the requirements needed to establish whether the right of priority was valid with respect to claimed subject-matter comprising features taken from drawings of a priority document. P1 and P2 each consisted of a description in text and drawings, but no claims. The board was not fully convinced that the skilled person would be able to clearly and unambiguously derive from P1 and P2 the function and structure of the display band region in the packages described in them, and found that the drawings did not show all the alternatives covered by the claim. More importantly, the packages of P1 and P2 addressed different problems from those addressed by the container of the disputed patent and had different combinations of features. The board concluded that neither P1 nor P2 disclosed the same invention as claim 1, as required by Art. 87(1) and 88 EPC. Therefore neither could serve as a basis for validly claiming a right of priority with respect to the subject-matter of claim 1.

3. First application – identity of invention

(CLB, II.D.4.1)

In case T 1525/12 the examining division had refused the then main request for lack of clarity. However, in some "further remarks not forming part of the decision" it had also concluded that claim 1 was not new in view of D1 (Art. 54(1) and (2) EPC), an earlier international application published between the priority date and the filing date of the application in suit. Although the examining division did not address the validity of the claimed priority in the decision itself, it referred there briefly to an earlier communication in which it had expressed the view that not the document from which priority was claimed, but D1, filed by the same applicant, was the "first application" disclosing the relevant subject-matter, and that therefore the priority claim was invalid.

According to the board, it could be assumed that the examining division had regarded D1 as prior art for this reason. The board referred to the "first application" as governed by Art. 87(1) and (4) EPC and by the corresponding provisions for international applications, Art. 4(A) and (C) Paris Convention in conjunction with Art. 8(2)(a) PCT. Since the filing date of the application published as D1 predated the filing date of the priority application of the application in suit, that application could theoretically qualify as the first application for the purposes of Art. 87(1) and (4) EPC and Art. 4(C) Paris Convention. However, this presupposed that the invention as claimed was directly and unambiguously disclosed in D1. The yardstick in this context was the same as for the assessment of novelty (see G 2/98, OJ 2001, 413; G 2/10, OJ 2012, 376). Features now added to the claims of the new main request found support in the document from which the application in suit claimed priority, but were not disclosed in D1. This document therefore could not be novelty-destroying for the subject-matter of the claims. Equally, and for the same reasons, the application published as D1 did not constitute, with respect to the subject-matter now claimed, the first application for priority purposes.

4. Multiple priorities for one claim – partial priority – application of G 1/15

(CLB, II.D.5.3)

In T 282/12 claim 1 related to a dosage form, the relevant feature of which was a gap "from 3% to 33%" formed by the coatings. The same range 3% to 33% was disclosed in the US priority document D1. The opponent challenged the novelty of claim 1 on the basis of an alleged public prior use during the priority interval of a product in which the gap width was approximately 17%. It contested the validity of the priority claim in view of an earlier application D22 of the proprietor (of which D1 was a continuation-in-part), which disclosed the range 5% to 33%. Therefore it was necessary to establish whether D1 or D22 was the first application within the meaning of Art. 87(1) EPC for the claimed subject-matter.

The board recalled the statement of the Enlarged Board of Appeal in G 2/98 (point 8.2 of the Reasons, OJ 2001, 413), that "in order to avoid any inconsistency, the criteria to be applied in assessing (i) whether an application is to be regarded as the first application for the purposes of determining priority and (ii) whether a claim in a later European patent application is in respect of the same invention as the priority application pursuant to Art. 87(1) EPC must be the same".

Applying those criteria in its comparison of D1 and D22, the board noted that the respective ranges 3% to 33% and 5% to 33% were not the same. It then referred to G 1/15 (OJ 2017, A82), in which the Enlarged Board acknowledged the concept of "partial priority" and confirmed its applicability to generic "OR"-claims. In the board's view, for reasons of consistency, the rationale of decision G 1/15 (concept of partial priority) must also apply in the context of deciding whether an application from which priority is claimed is the first application within the meaning of Art. 87(1) EPC. Indeed, just as a priority application and a patent claiming priority therefrom may partially relate to the same invention, the priority application and an earlier application filed by the same applicant may also do so. In that case, the priority application would be the first application in respect of only that part of the invention which is not the same as in the earlier application.

Accordingly, the board examined whether the invention disclosed in D22 was partly the same as that of D1. In line with the approach explained in point 6.4 of the Reasons of G 1/15, the board established that D22 disclosed the range 5% to 33%, that this range was encompassed in the range of D1, and that the invention of D1 was conceptually divided into two parts, one of which was the same invention as that of D22, i.e. the range 5% to 33%. For this sub-range D1 was therefore not the first application within the meaning of Art. 87(1) EPC, and the corresponding part of claim 1 was not entitled to the priority date of D1. This part of claim 1 was the relevant one in relation to the alleged prior-use product.

The board furthermore cautioned against assessing priority by performing a test under Art. 123(2) EPC, which approach had led the opposition division to find that D1 was the first application. The assessment of priority and of the requirement of Art. 123(3) EPC were both based on the concept of disclosure. However, applying a test under Art. 123(2) EPC could lead to wrong conclusions in certain circumstances because the concept of "partial" validity of an amendment does not exist, whereas "partial priority" does. The board agreed that amending a range from 5% to 33% to 3% to 33% would result in the addition of subject-matter. However, both of these encompassed an identical part, i.e. the sub-range 5% to 33%, which defined alternative dosage forms that did not change their identity, whether they were claimed as such or as part of a broader group of compositions, including other compositions with a gap width outside the range 5% to 33%. Thus, in the present case, the mere application of an assessment pursuant to Art. 123(2) EPC to the assessment of the validity of the priority claim did not allow the conclusion that D1 and D22 related in part to the same invention and therefore that the priority was not valid over the whole scope of claim 1.

In case T 260/14 the opposition division had decided that the patent was not entitled to the priority date claimed and that claim 1 lacked novelty pursuant to Art. 54(3) EPC in view of the priority document (D5) itself. The opponent's (appellant's) novelty attack was based on a working example disclosed in D5 (which was also disclosed in the description of the patent in suit).

The board applied Enlarged Board of Appeal decision G 1/15 (OJ 2017, A82), which ruled on the conditions for entitlement to partial priority for a claim encompassing alternative subject-matter by virtue of one or more generic expressions or otherwise (generic "OR"-claim). In its assessment, the board followed the steps set out by the Enlarged Board in point 6.4 of the Reasons. Thus, it first identified the above-mentioned working example as being relevant subject-matter disclosed in the priority document (cf. G 2/98, OJ 2001, 413). It then analysed whether claim 1 encompassed the working example, and more specifically whether the working example was alternative subject-matter by virtue of a generic "OR"-claim (with reference also to T 1222/11 and G 1/15, points 2 and 6.6 of the Reasons), by comparing the ambit of the claim with the content of the priority document (cf. G 1/15, point 2.4 of the Reasons and example decisions mentioned therein).

The claim was to a dental impression material comprising a base paste and a catalyst paste. Both pastes were described using generic features such as "polyethers" and "copolyether of ethyleneoxide". The working example was one specific embodiment of the claim. Multiple alternative working examples would be possible, with different variants falling within the generic features of the claim. The working example was thus alternative subject-matter by virtue of a generic "OR"-claim which fell within the ambit of claim 1. Hence, the part of claim 1 concerning that example was entitled to partial priority.

The divisional application in T 1519/15 had been refused for lack of novelty. The examining division had denied claim 1 the priority date of P1. The disclosure of P1 was limited to a specific circuit having four sensing capacitors, one mimicking capacitor and seven transistors, while claim 1 generalised the number of sensing capacitors and omitted the transistors and so the requirement of the same invention was considered not to be met. Accordingly, figures 4 to 6 of the parent application (D3), which were disclosed in P1, were held to belong to the state of the art under Art. 54(3) EPC.

The board applied the conclusion of the Enlarged Board in G 1/15 (OJ 2017, A82) in respect of the entitlement of a generic "OR"-claim to partial priority. Claim 1 included the generic expressions "a sensing circuit comprising at least first and second sensing capacitors" and "at least one mimicking capacitor". It did not specify whether there were any transistors. It thus encompassed the specific embodiment of figures 4 to 6 of D3, also disclosed in identical figures 4 to 6 of the divisional. Since this embodiment was disclosed in P1, claim 1 was entitled to the priority of P1 in respect of it. Figures 4 to 6 of D3 thus did not belong to the state of the art under Art. 54(3) EPC for the specific embodiment with four sensing capacitors falling within the ambit of claim 1.

The remaining (general) subject-matter claimed, i.e. in particular a sensing circuit having a number other than four sensing capacitors, only benefited from the priority date of P2. Those embodiments were not relevant to novelty under Art. 54(3) EPC. The disclosure of parent application D3 relating to a number of sensing capacitors other than four, see e.g. claim 1, was not entitled to the priority of P1.

The board set aside the decision and remitted the case for further prosecution.

E. Amendments

1. Article 123(2) EPC – added subject-matter

1.1 Gold standard and essentiality test

(CLB, II.E.1.2.4)

In T 1852/13 the board concluded that the essentiality test, developed for cases of replacement or removal of features from a claim as originally filed, should no longer be used: logic alone dictated that it could not be congruent with the "gold standard" (G 2/10, OJ 2012, 376; also known as the "disclosure test"). That also followed from the wording "may not" in decision T 331/87 (OJ 1991, 22), the board there itself having recognised that its three-part essentiality test might be met, yet Art. 123(2) EPC still infringed. Although the essentiality test could be a useful indicator in certain cases, the "gold standard" was the only test that counted (see T 755/12). Moreover the Enlarged Board, in G 2/98 (OJ 2001, 413), on the topic of when a priority claim was valid, had rejected the essentiality-based approach taken in T 73/88 (OJ 1992, 557; Snackfood), laying down instead a condition analogous to the "gold standard", and its concern that evaluations of essentiality might be arbitrary applied to amendments too. The board therefore agreed with T 910/03 that the conclusion to be drawn from G 2/98 was that the essentiality test should no longer be applied. Another advantage of the "gold standard" over the essentiality test was that it was a single test for all types of amendment and thus applicable to both deletions and additions.

The board in T 2599/12, after finding that that the amendment complied with the "gold standard", held that there was no need for any further investigation, such as applying the test laid out in T 331/87 (OJ 1991, 22). Ultimately, the essentiality test was meant to provide an indication of whether an amendment complied with Art. 123(2) EPC as interpreted according to the "gold standard". It did not take the place of the "gold standard" and should not lead to a different result than when applying the "gold standard" directly.

In T 782/16 the board held that for a correct application of the gold standard, a distinction needed to be made between subject-matter which was disclosed either implicitly or explicitly in the original (or earlier) application and therefore could be directly derived from it, and subject-matter which was the result of an intellectual process, in particular a complex one, carried out on what was disclosed. In the case at issue, the appellant's reasoning was based on an intellectual processing of the subject-matter disclosed in the original (or earlier) application rather than a direct and unambiguous derivation as required by the "gold" standard. No matter whether or not such processing was based on obvious considerations, such reasoning could not be used to justify the compliance with Art. 76(1) and 123(2) EPC.

1.2 Ranges – setting a new end-point with a value that has not expressly disclosed

(CLB, II.E.1.3)

In T 1986/14 claim 6 of the main request was amended by including the features "glycerin in an amount ranging from 50% to 90% by weight of the composition". The appellant argued that the amount of glycerin in claim 6 found a basis in the application as originally filed, which read "glycerin moisturizer can be present individually in an amount ranging from about 50.00% to about 90.00% by weight". The board held that it was undisputed that 50% and 50.00% differ in their accuracy. For this reason, 50.00%/90.00%, on their own, cannot provide a basis for the features 50% or 90%. The applicant argued, however, that the use of the term "about" in the passage mentioned above indicated that it was not intended to restrict the claimed amount to ranges defined by end-points with four significant figures. For the board, the feature "about 50.00% to about 90.00%" disclosed a range with two end-points, namely 50.00% and 90.00%, and an area of undefined boundaries around them. No other end-point, such as 50% or 50.0%, was either implicitly or explicitly disclosed. For this reason, the passage cited could not provide a basis for the aforementioned feature.

1.3 Disclaimers

(CLB, II.E.1.5)

In G 1/16 (OJ 2018, [ *** ]) the Enlarged Board considered that the choice of the proper test for assessing the allowability of any disclaimer is determined by the fundamental distinction, in terms of their legal nature, between disclosed disclaimers and undisclosed disclaimers. That distinction necessitates providing for each of the two classes of disclaimer a single specific test for assessing whether the introduction of a given disclaimer is in compliance with Art. 123(2) EPC. Therefore, for undisclosed disclaimers the proper test is whether the criteria of G 1/03 (OJ 2004, 413) are fulfilled, and for disclosed disclaimers the proper test is the gold standard disclosure test of G 2/10 (OJ 2012, 376).

The assessment of the allowability of a claim amendment by an undisclosed disclaimer is governed exclusively by the criteria laid down in G 1/03. No modifications are to be made to, nor any conditions added which go beyond, the criteria of G 1/03.

The Enlarged Board confirmed that an amendment by an undisclosed disclaimer may be allowable in the three situations mentioned in G 1/03 (point 2.1 of the order), i.e. in order to (1) restore novelty by delimiting a claim against state of the art under Art. 54(3) EPC; (2) restore novelty by delimiting a claim against an accidental anticipation under Art. 54(2) EPC; or (3) disclaim subject-matter which, under Art. 52 to 57 EPC, is excluded from patentability for non-technical reasons.

In addition, the undisclosed disclaimer must not be related to the teaching of the invention, as already held in G 1/03 (Order, point 2.3; point 2.6 of the Reasons). In endorsing this concept, the Enlarged Board provided the following explanations:

The question to be asked in this context is not whether an undisclosed disclaimer quantitatively reduces the original technical teaching – this is inevitably the case – but rather whether it qualitatively changes it in the sense that the applicant's or patent proprietor's position with regard to other requirements for patentability is improved. If that is the case, then the original technical teaching has been changed by the introduction of the disclaimer in an unallowable way. And as a consequence, the technical teaching based on the amended claim, i.e. on the remaining subject-matter without the disclaimer, can no longer be considered as belonging to the invention as presented in the application as originally filed.

For the sake of completeness, the Enlarged Board added that the prohibition of a qualitative change in the original teaching applies in an absolute way, i.e. not only with regard to the prior art which provides the basis for the undisclosed disclaimer, but also to the entire prior art relevant for the assessment of inventive step. In practical terms, this means that the evaluation of inventive step has to be carried out disregarding the undisclosed disclaimer, as proposed in T 710/92. In this way, any unallowable modification of the original technical teaching in the assessment of inventive step is avoided.

The Enlarged Board concluded that the introduction of an undisclosed disclaimer must fulfil one of the criteria laid down in point 2.1 of the order of decision G 1/03, but may not provide a technical contribution to the claimed subject-matter of the application as filed; in other words, the identity of the invention as originally filed must remain unchanged by the subject-matter remaining in the claim after the introduction of the undisclosed disclaimer.

The Enlarged Board answered the questions referred to it as follows:

For the purpose of considering whether a claim amended by the introduction of an undisclosed disclaimer is allowable under Art. 123(2) EPC, the disclaimer must fulfil one of the criteria set out in point 2.1 of the order of decision G 1/03.

The introduction of such a disclaimer may not provide a technical contribution to the subject-matter disclosed in the application as filed. In particular, it may not be or become relevant for the assessment of inventive step or for the question of sufficiency of disclosure. The disclaimer may not remove more than necessary either to restore novelty or to disclaim subject-matter excluded from patentability for non-technical reasons.

2. Article 123(3) EPC

2.1 Extent of protection

(CLB, II.E.2.3)

In T 1896/11 the appellant submitted that, taking Art. 69 EPC into account, the description had to be used to interpret the claims; since the protection conferred by claim 5 of the patent as granted necessarily extended to what was stated in the description, claim 5 could be corrected to reflect the description without infringing Art. 123(3) EPC. However, the board held that, in the case at issue, the description could not be used to give a different meaning to a claimed method step which in itself imparted a clear, credible, technical teaching to the skilled reader. Otherwise third parties could not rely on what the claim actually stated. In order to determine whether the claimed method step in itself imparted a clear, credible technical teaching, it was necessary to examine "whether (a) the step as claimed is in itself meaningful and plausible from a technical point of view, and (b) there is, prima facie, any inherent incompatibility with the remaining features of the claim" (see decision T 1202/07). In the case in hand, the relevant feature was meaningful and plausible from a technical point of view.

F. Divisional applications

1. Inadmissible extension – amending the description

(CLB, II.E.1.12)

The board in T 835/11 held that an amendment to the description could result in an inadmissible extension only if it changed the subject-matter. That might be the case, for instance, if the description initially defined a feature in the claims more narrowly than its usual meaning; deleting the definition from the description could then result in an inadmissible extension of the patent's subject-matter. Removing or adding examples might also affect how the claims were understood and so also change the patent's subject-matter (see e.g. T 1239/03). The boards had also repeatedly held that reformulating the problem to be solved could inadmissibly extend the patent's subject-matter (see e.g. T 13/84, OJ 1986, 253). By contrast amendments to the description which had no effect on the claimed subject-matter were not open to objection. In the case in hand, Art. 100(c) EPC had not been infringed.

2. Language for filing divisional application

(CLB, II.F.2.1.3)

According to R. 36(2), first sentence, EPC, a divisional application shall be filed in the language of the proceedings of the earlier application. According to R. 36(2), second sentence, EPC, if the latter was not in an official language of the European Patent Office, the divisional application may be filed in the language of the earlier application; in such a case, a translation into the language of the proceedings for the earlier application shall be filed within two months of the filing of the divisional application.

In J 13/14 the Legal Board observed that a divisional application of an earlier application, which was filed in an EPO official language, had also to be filed in the EPO official language of the earlier application. Otherwise, it would have been filed in an inadmissible language. In this case, a correction of the language deficiency by means of a translation into the language of the proceedings for the earlier application is neither required under R. 36(2), second sentence, EPC nor would it even be admissible in view of the wording of that provision and of G 4/08 (OJ 2010, 572). It was not possible for the applicant to remedy the language deficiency in its divisional application either by means of a correction under R. 139, first sentence, EPC, or by means of an amendment under Art. 123(2) EPC. The legal consequence of the non-compliance with the language requirements was that the divisional application could not be treated as a valid divisional application by analogous application of Art. 90(2) EPC.

3. Pendency of earlier application

(CLB, II.F.3.4.3)

In J 10/16 the board found that a divisional application could not be filed from the point in time at which the parent application was deemed to have been withdrawn (here: after expiry of the six-month period under R. 161(1) EPC) and if the applicant did not act on the communication noting this loss of rights under R. 112(1) EPC. Where, after receiving such a communication, the applicant did not apply for a decision under R. 112(2) EPC, the rights were lost on expiry of the original unobserved time limit (see e.g. J 4/86, OJ 1988, 119; G 1/90, OJ 1991, 275; G 4/98, OJ 2001, 131; J 19/01; J 9/02); the communication became final; and the proceedings came to an end also on expiry of the original time limit – unless the legal effect in question was set aside by either further processing or re-establishment of rights. Communications under R. 112(1) and decisions under R. 112(2) EPC were purely declaratory findings of a loss of rights already having arisen by operation of law (see J 1/05).

III. RULES COMMON TO ALL PROCEEDINGS BEFORE THE EPO

A. The principle of the protection of legitimate expectations

1. Contradictory acts

(CLB, III.A.2.3)

In T 1825/14, the examining division had set a time limit for filing observations and correcting indicated deficiencies. The applicant requested an extension of the time limit. The examining division granted this request and the appellant filed the response within the extended time limit. Subsequently, however, the examining division issued a communication noting a loss of rights pursuant to R. 112(1) EPC. The reason given was that the invitation to file observations had not been complied with in due time, as the observations had not been filed within the original time limit. The applicant was informed that the request for extension of the time limit had been granted "erroneously" as it had been filed one day late.

The board noted that the request for an extension of the time limit had initially been granted by the examining division and had thus been treated as a valid procedural act. Relations between the EPO and applicants are governed by the principle of good faith (J 2/87, OJ 1988, 330 and J 14/94, OJ 1995, 825). Hence, if the EPO issues a communication such as a receipt of a response within a set time limit, and then treats that response as a valid procedural act, it cannot subsequently go back on its own earlier position, especially when, as here, it had led the applicant to legitimately believe that no loss of rights had taken place. The examining division was thus prevented by its own earlier conduct from issuing the notice of loss of rights.

B. Right to be heard

1. Surprising grounds or evidence

(CLB, III.B.2.3)

In the opposition appeal underlying review proceedings R 7/15, the technical board had decided that subject-matter of claim 1 of the main request and of the auxiliary request extended beyond the content of the application as filed (Art. 100(c) EPC). The proprietor alleged a fundamental procedural violation of its right to be heard (Art. 112a(2)(c) and 113(1) EPC) in that the board's decision was based on grounds forming together a new line of argumentation and on new evidence, both absent from the oral debate before the board. In particular, the board had based its reasoning on assumptions concerning the skilled person's technical knowledge as regards the topical and oral administration of Apremilast for the prevention and/or treatment of psoriasis. These allegedly erroneous assumptions had led to the finding that the requirements of Art. 123(2) EPC were not met.

The Enlarged Board rejected this argument. From its analysis of the board's reasoning, it considered that the board had adopted the usual way of assessing whether the features of a claim fulfil the requirement under Art. 123(2) EPC for a clear and unambiguous disclosure in the application as filed taken as a whole. It was not correct to contend that the board had made "assumptions" relating to the knowledge of the skilled person. The board had rather concluded from the information on file that another construction of the claim than the one developed by the petitioner could have been considered by a skilled reader and had therefore drawn the conclusion that claim 1 did not pass the disclosure test.

It followed that the board did not introduce a new ground in their decision in writing. In this respect, the Enlarged Board was following the settled case law, which draws a distinction between a ground and an argument (see R 15/12). In the case now in issue, the report of the oral proceedings before the board submitted by the petitioner showed that the ground for opposition under Art. 100(c) EPC had been extensively debated, and that the chairman had guided the discussion so as to emphasise a possible difference between treatment and prophylaxis, with successive questions put to the proprietor. Moreover, the chairman had explained, in response to the proprietor's enquiry, that the board's finding related to the selection of treatment or prevention and thus a selection from two lists, and the proprietor had then indicated that he was not willing to amend the claim in order to add "prevention" to "treatment".

The Enlarged Board therefore concluded that no new ground was introduced into the debate by the board on which the petitioner did not have the opportunity to comment before the decision was reached. What formed the core of the petition were arguments made by the board in respect of how the skilled person would have understood the claim, taking into account the content of document D7 (filed by the proprietor to illustrate the common general knowledge). It was settled case law that the board had no obligation to inform the parties about all the arguments supporting the reasoning of the decision. The requirement of Art. 113(1) EPC did not extend that far (see for instance R 15/13). Lastly, it was obvious that no new material had been introduced into the debate as evidence since document D7 had been part of the file from the start of the opposition proceedings.

1.1 The meaning of "grounds or evidence"

(CLB, III.B.2.3.2)

In T 556/15, the examining division's impugned decision was based entirely on a lack of compliance with Art. 123(2) EPC. The board's comparison of the objections under Art. 123(2) EPC raised by the examining division in its two communications with those forming the grounds for its decision revealed that the latter had come as a surprise to the appellant. It had never had a chance to comment on those grounds, having only become aware of the new objections under Art. 123(2) EPC on receiving the decision.

In the board's view, the term "grounds or evidence" in Art. 113(1) EPC should not be interpreted narrowly but rather within the meaning of T 951/92. There, the word "grounds" had been found not to refer merely to a ground for objection to the application in the narrow sense of a requirement of the EPC which was considered not to be met, e.g. Art. 123(2) EPC in this case. "Grounds" should rather be interpreted as referring to the essential reasoning, both legal and factual, which led to refusal of the application. In other words, before a decision was issued an applicant had to be informed of the case which it had to meet, and had to have an opportunity to meet it.

This case differed from that dealt with in T 951/92 in so far as the communications sent by the examining division to the appellant contained objections that, while detailed, did not concern any of the matters on which the decision was ultimately based. The appellant had thus been unaware that the features of the claims under discussion had violated Art. 123(2) EPC until it received the decision. The fact that the appellant had had several opportunities to amend the claims before the decision was irrelevant. What mattered was that the appellant had not had a chance to comment on the grounds for the decision.

2. Text submitted or agreed by applicant (patent proprietor) – Article 113(2) EPC

2.1 General

(CLB, III.B.3.1)

In T 1440/12 the board held that in the case of opposition, the intention behind Art. 113(2) EPC 1973 is that the EPO may not maintain a patent according to a particular text unless the proprietor has consented unambiguously to the patent being maintained in that form. The "text submitted" is to be understood to mean a text submitted by the proprietor with the clear intention that the patent be maintained according to that text, at least as an auxiliary measure. In the case in issue, although six new requests were enclosed with the reply to the statement of grounds of appeal, the proprietor/respondent did not actually request maintenance of the patent on the basis of any of them, but merely described them as "six auxiliary requests that the proprietor may subsequently choose to rely upon". This phrase, although the submissions in question were termed "requests", made it clear that the proprietor was not at that point requesting maintenance of the patent based on them, but merely leaving open the possibility that it might choose to make such a request subsequently. In practice, it did not. In the absence of any clear request, the board was not empowered to decide whether the patent should be maintained according to the "six auxiliary requests".

At the end of the oral proceedings in T 1477/15, when the chairman asked for the parties' final requests, the patent proprietor maintained its main request, new auxiliary request 3 and auxiliary request 9, but withdrew auxiliary requests 1 and 2 filed with its statement setting out the grounds of appeal. Since auxiliary request 2 was the request on which the decision under appeal was based, the opponents took the view that they had a right to a substantiated decision on their successful appeal against the impugned decision, so the patent proprietor did not have the right to withdraw this request. The board failed to see any basis in the EPC for not allowing the patent proprietor to withdraw its auxiliary requests 1 and 2 (Art. 113(2) EPC). The board further stated that it is generally accepted that in appeal proceedings the principle of party disposition applies (see e.g. R 13/13, point 15 of the Reasons), meaning that parties can put forward, withhold or withdraw their requests as they see fit. In other words, if a patent proprietor withdraws or no longer agrees to a text (two auxiliary requests, in this case), this principle prevents the board of appeal from deciding on these issues.

2.2 Cases where the EPO is uncertain or mistaken about the approval of the text

(CLB, III.B.3.3)

In T 1104/14 the board stressed that one of the fundamental principles of European patent law was that responsibility for the wording of patent claims and the filing of any associated request rested solely with the applicant or patent proprietor. It also noted that a distinction had to be made between the act of filing a request and the question of whether the request could be refused as late-filed in the decision. According to the boards' established case law, the EPO was not obliged under Art. 113(2) EPC to admit a request for amendment put forward by the applicant (see G 7/93, OJ 1994, 775). This however had to be understood to refer only to the decision on whether to admit the (filed) request; it did not mean the boards had the power to refuse to permit the filing of a request or not to record or accept a request during oral proceedings. That would generally constitute a serious breach of the autonomy granted by Art. 113(2) EPC to the applicant or patent proprietor to determine the text of its patent ("principle of free party disposition").

In the case in hand, the opposition division had disregarded the autonomy enjoyed by the proprietor (appellant) under that principle by failing to base its decision on the requests that, according to the minutes, it had actually submitted for decision in the oral proceedings. It made no difference that those requests might have been unclear. Any doubts in relation to the requests referred to in the minutes, e.g. which were to replace which, were for the chairperson to clarify.

Failure to obtain clarification where needed also amounted to a procedural violation because it was then not clear which version of the patent was being put forward by the patent proprietor for decision, resulting in a breach of Art. 113(2) EPC. Basing a decision on the wrong requests constituted a substantial procedural violation because freedom of disposition was cardinal and disregarding it adversely impacted the entire proceedings.

C. Oral proceedings

1. Late submission of new facts and evidence in the preparation for oral proceedings – Rule 116 EPC (Rule 71a EPC 1973)

(CLB, III.C.4.4)

In T 1750/14 the board interpreted R. 116(1) EPC in conjunction with R. 132(2) EPC. From the wording of R. 116(1) EPC it was not clear whether the non-applicability of R. 132(2) EPC to R. 116(1) EPC also precluded the possibility of extending specified periods, implying that the final date for making submissions in preparation for the oral proceedings set under R. 116(1) EPC could not be changed. The board considered that a change of the final date should normally be allowable when the date for oral proceedings is postponed. At least in cases where a final date is specified relative to the date for oral proceedings (usually one month before the scheduled oral proceedings), it could even be argued that the final date was postponed automatically when the oral proceedings were postponed.

2. Doubt as to the nature of the request for oral proceedings

(CLB, III.C.2.1)

In T 1972/13 the board held that the applicant's main request before the examining division for continuation in writing, with a subsidiary request for oral proceedings, although imprecisely worded, did not embrace the issuing of an immediate decision without oral proceedings being held. It was not relevant that oral proceedings had been initially arranged ex officio and not in response to a request by the applicant.

D. Re-establishment of rights

1. Right to file request for re-establishment of rights

(CLB, III.E.2)

In T 181/14, the appellant (opponent), having filed notice of appeal and a statement of grounds of appeal but failed to pay the appeal fee in due time, was seeking re-establishment of its rights in respect of the time limit for payment. The board saw no reason to depart from the settled case law that Art. 122(1) EPC was applicable only where an appellant-opponent had failed to observe the time limit for filing its statement of grounds of appeal. When it came to re-establishment of rights, there was good reason to treat such a failure to file the statement of grounds of appeal differently from late payment of the appeal fee. Appellants-opponents who failed to observe the time limit for filing the statement of grounds of appeal could have their rights re-established because there was an appeal having legal effects; that is to say, appeal proceedings had been validly initiated. By contrast, if the appeal fee was not paid on time, there was no appeal. It made no difference, the board held, that, in this case, the patent proprietor had replied to the statement of grounds of appeal.

2. Loss of rights as a direct consequence by virtue of the EPC

(CLB, III.E.3.4)

In J 23/14 the Legal Board held that Art. 122(1) EPC was worded such that it was applicable only where the non-observance of the time limit in question had the direct consequence of a loss of a right or of a means of redress. Thus, according to established jurisprudence, re-establishment of rights could be granted only with respect to the time limit pursuant to R. 51(2) EPC for paying the renewal fee with an additional fee.

This jurisprudence was based on the view that the non-payment of the renewal fee by the due date pursuant to R. 51(1) EPC did not result in a loss of rights but that the direct consequence of a loss of rights occurred on expiry of the time limit under R. 51(2) EPC if it had not been observed. However, the Legal Board was also aware of decision T 1402/13 of 31 May 2016, in which the board held that, whereas under Art. 86(3) EPC 1973 the loss of rights did not occur before the additional period of six months had elapsed, under the now applicable Art. 86(1) EPC 2000 an application was deemed to be withdrawn if the renewal fee was not paid in due time pursuant to R. 51(1) EPC. The board in that case further stated that the due date according to R. 51(1) EPC for paying the renewal fee was "not a time limit in the narrow sense of the word" and that R. 51(2) EPC provided for a remedy, namely the possibility that a deemed withdrawal could be reversed if the annual fee and the additional fee were paid within six months after the due date.

The Legal Board concluded that this finding in decision T 1402/13 implied that restitutio in integrum in respect of the time limit specified in R. 51(2) EPC would no longer be admissible since, according to the reasoning of said decision, non-compliance with this time limit would not have "the direct consequence of causing a loss of rights" as required by Art. 122(1) EPC.

However, this issue had been clarified in the amended version of R. 51(2) EPC resulting from the decision of the Administrative Council of 14 December 2016. A second sentence has been added to R. 51(2) EPC, which reads as follows: "The legal consequence laid down in Article 86, paragraph 1, shall ensue upon expiry of the six month period." Although this new provision only applied as of 1 January 2017, it clearly confirmed that – despite the wording of Art. 86(1) EPC 2000 – it had not been the intention of the legislator to cancel re-establishment of rights as a means of redress in such cases.

Hence, the Legal Board held that, notwithstanding the wording of Art. 86(1) EPC 2000 and R. 51 EPC (in the version in force until 31 December 2016), and for the sake of the protection of legitimate expectations of the users of the European patent system, a patent application was deemed to be withdrawn only upon expiry of the six-month grace period for paying the renewal fee with additional fee under R. 51(2) EPC, in accordance with the case law that prevailed before decision T 1402/13.

3. Admissibility of requests for re-establishment of rights

(CLB, III.E.4)

In T 1588/15 the board dismissed an appeal on a decision rejecting a request for re-establishment of rights. In the case in hand, the admissibility of the request turned on the question of the removal of the cause of non-compliance mentioned in R. 136(1) EPC. The decisive single issue was whether this had occurred upon receipt of the notification of loss of rights by the representative, or rather when this had been forwarded to the appellant.

The appellant's submission implied a change in the case law entailing acceptance that the receipt of the notification of the loss of rights by a registered representative should not remove the cause of non-compliance under certain, but otherwise "normal" (because not special) circumstances. This argument was also based on the efficiency pressure imposed on IP teams. The appellant argued that fee payment through specialised third-party firms was widespread, and in such cases the registered representative could not be expected to know if the loss of rights was intended or not. The board rejected this argument. It held that an applicant could not be considered to act with all due care if it regularly did not inform a representative if and when applications were abandoned. Even when a representative was explicitly exempted from looking after a fee payment, at least he could be expected to be informed at all times about the applicant's intention as to whether or not the application was to be maintained. Only in this manner was a professional representative capable of acting in the best interests of his client, which was his professional duty. He had a particular duty to know the status of the cases handled by him, which was derivable from his duty to be able to inform his client in this respect (see point 1, paragraph (c) and point 4, paragraph (a), of the epi Code of Conduct). An applicant could decide to keep his representative uninformed, but then had to be prepared to bear the consequences. In short, if the representative was indeed regularly uninformed of any possible withdrawal or abandonment of the appellant's applications, this established a lack of due care on the part of the appellant.

The board saw no good reasons for the representative being uninformed and not taking immediate action in the knowledge (or merely belief) of being uninformed. In this respect, the finding of the board differed from the line taken by decision J 1/13, where the deciding board accepted that the cause of non-compliance was not removed when the noting of loss of rights was received in the offices of the representative, because she had good reasons to believe that the applicant wanted to abandon the application, in light of the fact that simply not instructing the representative was a usual way of abandoning applications. In that case the deciding board appeared to have tacitly accepted that such a belief on the part of the representative would be compatible with the due care expected from her. A specific duty of an applicant to positively inform its representative about an intended abandonment of an application was not examined in J 1/13. Accordingly, the board confirmed the examining division's finding that the request was filed late, because at least the appellant, but possibly also the representative had not shown all due care in, respectively, sending instructions and forwarding official mail in relation to renewal fees.

E. Languages

1. Translations

(CLB, III.F.5)

In T 1332/12 the opposition division had decided that claim 1 of the main and several auxiliary requests lacked an inventive step over Japanese application D7 and the common general knowledge, its analysis being based on D7T, a JPO machine translation of D7 into English filed by the opponent.

During the appeal proceedings the patent proprietor filed D7JPO, a later and, in its view, more accurate machine translation of D7.

The board pointed out that there is nothing in the EPC to prevent a party from filing a corrected translation of a document filed as evidence, even if the evidence and/or translation was filed by the other party to the proceedings. In its view, this also applied if the document was a patent application, taking into account that, under the EPC, the translation of a European patent application or an international application into an official language of the EPO may generally be brought into conformity with the application as filed (see Art. 14(2) EPC 1973 regarding European patent applications; T 700/05 and T 1483/10 regarding international applications). Hence, the board considered that the translation into English of Japanese prior-art document D7 could be brought into conformity with the original. Moreover, the differences between the two translations were minor and resulted in a linguistic clarification of certain passages in D7T without changing its technical disclosure. Consequently, the board decided to admit translation D7JPO into the appeal proceedings and base the discussions of inventive step on D7 with reference to it.

F. Law of evidence

1. Taking of evidence

1.1 Time frame for submitting evidence

(CLB, III.G.3.2)

The board in ex parte case T 545/08 stated that when exercising its discretion in the case in hand, the examining division should have been aware that, although it had cited document D1 (internet publication from a commercial website) at the beginning of the examination procedure, it had never provided any further explanations or evidence as to the document's public availability before the priority date, meaning that the objections based on this document had not been properly raised. Moreover, the arguments provided by the examining division for the exercise of its discretion were not persuasive. Under these circumstances, the examining division was not allowed to reject the appellant's further evidence as late-filed and to continue to rely on document D1 as prior art. It thereby infringed the appellant's right to be heard.

2. Evaluation of evidence

2.1 Witness testimony and written statements

(CLB, III.G.4.2.1)

In T 2057/13 the board did not admit into the proceedings the affidavit in respect of the contents of the priority document P1, filed for the first time by the patent proprietor in reply to the board's communication, since it was not prima facie relevant for assessing the knowledge of the skilled person when reading P1. The affidavit had been written by one of the inventors named in P1 and an employee of the appellant (patent proprietor). The board had to assess the contents of P1 in an impartial manner from the perspective of an independent skilled person.

In T 1107/12, the respondent (patent proprietor) disputed that a document cited in opposition had been publicly available before the priority date of its patent, arguing, in essence, that the testimony taken from a witness on this point had not established to the high degree of certainty required by the case law that it had actually been distributed to potential customers of one of the appellants.

The board could find nothing to suggest the opposition division had erred in law when evaluating the evidence. In particular, there was no indication that it had applied the wrong standard of proof, even if it had not explicitly discussed whether the strict standard of "beyond reasonable doubt" or the usual "balance of probabilities" standard should be applied in this case. The case law requiring absolute certainty or proof beyond all reasonable doubt which the respondent had cited in support of its position had been concerned with alleged public prior use (see T 441/04, T 472/92, OJ 1998, 161 and T 2451/13) and had applied that standard because, in such cases, the supporting evidence was almost always in the opponent's possession.

The case in hand turned on whether a document originating from one of the opponents had been publicly available on a particular date. Its availability was supported by witness testimony, the probative value of which was to be evaluated freely. Ultimately, it could be left open whether the stricter standard of proof had to be applied in such circumstances. The opposition division was convinced that there was no reason to doubt the witness had been telling the truth. Its evaluation of the evidence was not otherwise vitiated by any error in law. It was based on the right criteria, could be followed in all respects and did not contain any logical mistakes. It was therefore not open to the board to overrule its findings and re-evaluate the evidence in its place.

2.2 Archives and internet publications

(CLB, III.G.4.2.3)

In T 1711/11, the cited prior art included a document with four pages showing seven screenshots of website pages stored in the internet archive at www.archive.org and retrieved using its "Wayback Machine". The board, however, found that the case law cited by the appellant (T 1134/06, T 1213/05 and T 1875/06) was obsolete: in their more recent case law, the boards had unanimously considered the "balance of probabilities" standard sufficient and expressly abandoned the line taken in e.g. T 1134/06 (see T 286/10, which contains detailed reasoning citing the now applicable case law, as established in T 2339/09 and T 990/09; see also T 2227/11). There was thus no legal basis for subjecting internet disclosures to a different standard of proof from that generally applicable to prior-art disclosures. The opposition division's evaluation of the evidence was in line with the boards' case law, the EPO's notice concerning internet citations (OJ 8-9/2009, p. 456-462) and the EPO Guidelines for examination relating to such disclosures.

In ex parte case T 545/08, the contested decision was based on the assumption that document D1, an internet disclosure, belonged to the prior art. In the board's view, the crux of the matter was adequately summarised in the Guidelines G-IV, 7.5.1, second paragraph (November 2016 – unchanged in the November 2017 version of the Guidelines). T 1134/06 applied the strict standard of proof "beyond any reasonable doubt" (see also T 373/03, T 1875/06, T 19/05). Decisions T 286/10 and T 2227/11 applied the yardstick of the balance of probabilities (see also T 990/09, T 2339/09). The board in the case in issue stated that it was no easy task to try to reconcile the different views. However, any such attempt had to respect the European Patent Convention's overarching principle of free evaluation of evidence (see decision G 1/12, OJ 2014, A114). The question whether a fact could be regarded as proven had to be assessed on the basis of all the relevant evidence (G 3/97, OJ 1999, 245). In the board's view, the standard of the "balance of probabilities" should not be misunderstood as implying that a fact has to be regarded as already proven when it has a probability which is "just tipping the balance slightly", for instance, a probability of 51% The board concluded that it was correctly stated in the Guidelines (G-IV, 7.5.2): "The standard for assessing these circumstances is the balance of probabilities. According to this standard, it is not sufficient that the alleged fact (e.g. the publication date) is merely probable; the examining division must be convinced that it is correct."

2.3 Standard of proof

2.3.1 Receipt of formal documents

(CLB, III.G.4.3.6)

In T 1/12, the appellant (opponent) had filed notice of opposition on the last day of the opposition period but it was not possible to identify from that notice who had actually filed it. The appellant, however, contended that it had also filed EPO Forms 1010 and 2300 together with its notice. According to the boards' settled case law (in particular G 1/12, point 31 of the Reasons, OJ 2014, A114), the principle of free evaluation of the evidence applies in EPO proceedings. Applying that principle, the board concluded that Form 2300 had been filed on time, together with Form 1010, because the appellant's evidence showed that this was considerably more likely than not (J 20/85, OJ 1987, 102 and T 1200/01). It was established that at least one page of the documents filed by the appellant on the last day of the opposition period had gone missing at the EPO, and that alone showed it was likely that other pages had been mislaid too. The evidence adduced included a confirmation copy of Form 1010 bearing an EPO stamp of that date, statements made by an employee of the appellant's professional representative and traces left by staples. The board therefore held that the EPO had received Form 2300, which bore the appellant's name as opponent, within the opposition period and that, therefore, the opposition met the requirements under Art. 99(1) EPC and was admissible.

G. Main and auxiliary requests

1. Request not sufficiently defined

(CLB, III.I.3.1)

In T 1138/12, as a second auxiliary request the appellant (patent proprietor) asked the board to maintain the patent on the basis of those claims in its first auxiliary request that it considered grantable. The board decided that this request was insufficiently defined and therefore could not be admitted into the proceedings. The reason was that, under Art. 113(2) EPC 1973, the board had to examine and decide on a patent only in the text submitted by the proprietor. Under Art. 101(3) EPC, amendments proposed by the proprietor had to be examined to decide whether the patent as amended met the requirements of the EPC or not. Where, as was the case here, the proprietor proposed an amendment in a first auxiliary request, in other words requested that the patent preferably be maintained in that amended version, and any one of the claims did not meet any of the requirements of the EPC, there was no basis in the EPC for examining the patentability of the other claims of the request because the patent could not be maintained according to that request anyway. In the absence of a legal basis for continued examination, the condition according to the second auxiliary request for determining its subject-matter, and thus a specific set of claims on the basis of which the patent could be maintained – namely that the board had identified the grantable claims of the first auxiliary request – could not be met.

2. Withdrawal of request

(CLB, III.I.)

In T 388/12 the opponent argued before the board that auxiliary request 4 had been withdrawn by the patent proprietor in the opposition proceedings and was, therefore, no longer pending before the board. The board held that no intention to withdraw former auxiliary request 4 could be recognised. As a general principle of law, surrender of a right could not be simply presumed (with reference to G 1/88, OJ 1989, 189). Relying on a strict application of the principle "a jure nemo recedere praesumitur", the withdrawal of a request could only result from acts of the party that manifestly establish such intention. Explicit withdrawal of a request would not be required in so far as the intention of the party, as it might result from its behaviour or comments made, was unequivocal.

H. Suspected partiality

(CLB, III.J.3.4)

In T 792/12 the appellant contended that the board did not want to listen to his arguments and appeared to lack impartiality because it interrupted him on several occasions. The board rejected this contention and referred to Art. 15(4) RPBA, according to which the chairman of the board presides over the oral proceedings and ensures their fair, orderly and efficient conduct. According to the board, a chairman may interject in a party's submissions to ensure that the proceedings are efficiently conducted, in particular to avoid a party repeating arguments. In the same way, a chairman, or indeed any member of the board, may interrupt to ask questions which are, for example, considered important for reaching a decision.

I. Formal aspects of decisions of EPO departments

1. Opposition division

(CLB, III.K.2.2)

In T 1049/11 the appellant requested the board to order that different examiners be allocated to this case after remittal because the first-instance proceedings had been hampered by the examiners' ineptness. The board pointed out decision T 2111/13, according to which the boards of appeal do not have any power regarding the designation of the division responsible for the further prosecution of the case after remittal. However, even if the board had the necessary power, the board believed that there would be no compelling case here for ordering a changed composition. While the examining division made errors of judgement, such errors could not, in the case in hand, be a sufficient reason to order a changed composition of the examining division.

2. Completion of the internal decision-making process

(CLB, III.K.3.2)

In T 1138/12 the board noted that the EPC did not prescribe for how long an opposition division had to deliberate to reach a decision. It was therefore conceivable, for example, that the members, having discussed the case before the oral proceedings, might find their preliminary opinion confirmed by the parties' submissions at those oral proceedings and then agree that that was the case by, for instance, simply making eye contact or nodding at the end of the discussion with the parties. Both the duration and the form of an opposition division's deliberations depended on the scope and complexity of the case at hand, the degree of compatibility of the members' opinions, which might diverge, playing a major role in this regard. But there were no binding rules on divisions' deliberations.

3. Department of first instance issues more than one decision

(CLB, III.K.4.1.4)

In T 1972/13 the applicant's request for a refund of an additional search fee was refused in a second decision issued by the examining division. Before the second decision was issued, an appeal had been filed against the first decision. The board considered that the first decision refusing the application terminated the proceedings before the examining division. After notification of the decision, the examining division had no power to go beyond a correction of errors in the decision or, considering that an appeal had been filed, granting interlocutory revision. It followed that the second decision taken by the examining division after the notification of the first one, and a fortiori after the filing of the appeal, was ultra vires and thus without any legal effect. The second decision was therefore null and void.

The board noted that it was not relevant whether a separate appeal against the second decision had been filed, given that an appeal against a legally void decision could logically have no legal effect (T 1257/08).

4. Examples of non-compliance with the requirements of Rule 111(2) EPC

(CLB, III.K.4.2.3)

In J 18/16 the appealed decision merely referred to a preceding communication on deficiencies in the application, and the section of the form headed "Further comments on the remaining deficiency" gave no information on why the Receiving Section had considered the subsequently filed documents not to have corrected the deficiencies noted or to what extent those documents made amendments going beyond what would have been sufficient to correct them, thereby infringing R. 58, second sentence, EPC. That, the board held, breached R. 111(2) EPC, according to which an appealable decision by the EPO had to be reasoned, a requirement that enabled the appellant and the board to assess whether the decision was justified or not. Pursuant to Art. 11 RPBA, the board remitted the case to the department of first instance on account of a substantial procedural violation.

5. Decisions according to the state of the file

(CLB, III.K.4.3)

In T 737/11 the applicant had withdrawn its request for oral proceedings and requested a decision on the state of the file. Nevertheless, oral proceedings were held in the absence of the applicant and a decision was taken at the end, using the standard form. This referred to two previous communications. It did not, however, mention the oral proceedings or the minutes of the oral proceedings. The board concluded that in the case in issue considerable uncertainty existed as to the precise reasons on which the contested decision was based. Since the written reasoning of the contested decision failed to mention the oral proceedings and to refer to their minutes, it was not possible to interpret the reasons for the decision in the light of these minutes. Furthermore, the claims had been amended substantially between the two communications to which the written reasoning of the contested decision referred. The board therefore found that a substantial procedural violation had taken place.

J. Notification

1. Forms of notification

(CLB, III.O.1)

In T 1693/13, the EPO had notified the contested decision using courier service UPS rather than by post. It had done so before 1 April 2015, when a new version of R. 126 EPC, amended by Administrative Council decision CA/D 6/14 of 15 October 2014 to read "postal services" and "postal service provider" in place of "post" of "posting", had come into force. The board considered that, in these circumstances, the appellant had been entitled to assume that the UPS courier service could be regarded as "post" within the meaning of R. 126 EPC as then still in force, and so to rely in good faith on its making no difference to how the appeal period was determined under that provision whether a decision had been sent by post or by UPS. The board based its finding on the legitimate expectations of parties as to the conduct of EPO organs. It followed from the protection of those legitimate expectations, a principle recognised by the boards and the Enlarged Board of Appeal, that parties had to be entitled to rely in their dealings with the EPO on due application of the law and on the lawfulness of related procedural conduct.

2. Spheres of risk and apportioning the burden of proof

(CLB, III.O.4)

In T 1934/16 the appellant denied having received the communication under R. 82(3) EPC, so the EPO had to establish that it had reached its destination and the date on which it had been delivered (R. 126(2) EPC). Following the enquiry, the letter in question was found to have been delivered to the law firm of the appellant's representative. The letter had a registered number and contained a reference to the patent's application number and could therefore be identified. After being informed of the result of the enquiry, the appellant did not supply any substantive response, such as facts or arguments casting doubt on the result. In such circumstances, the evidence of the receipt produced by the enquiry was found to be reliable and complete for proving the proper delivery of the letter. This does not contradict the decisions J 9/05 and J 18/05, where a confirmation letter by the postal service was held not to be sufficient, because in those cases the appellant had filed a considerable amount of counter-evidence and pointed out specific reasons why the letter might not have been received by the representative's office.

In T 2054/15 the board decided that pursuant to R. 126(2) EPC, in the event of a dispute, it is incumbent on the EPO to establish the date on which a letter was delivered to the addressee. However, the party seeking application of this legal provision had to set out the facts justifying it. The burden of proof on the EPO could not be taken to mean that the party was under no obligation to help clarify the circumstances within its own sphere of responsibility. The EPO was liable for risks arising both in its own sphere and during "transport", but the recipient was liable for those within its own sphere of organisation and influence (e.g. T 1535/10). When a registered letter had been delivered to the address of the representative, it was incumbent on him to establish that the letter had not been received by a person authorised to take delivery of it, or did not actually reach him for some other reason. In the case in issue, the EPO had established that the letter was delivered to the address of the representative. The board considered that the appellant had not discharged his burden of showing why the letter did not reach him personally once it was within his own sphere of control. The mere indication that the person who signed for it was not his employee and that he did not know her was not sufficient in that respect. No further declaration or other evidence had been provided. On the contrary, it appeared that a further notification sent by the EPO to the representative at the same address, of which the acknowledgement of receipt was also signed by the same person, did indeed reach him without any problem.

K. Representation

1. Legal practitioners entitled to act as professional representatives

(CLB, III.R.2)

In T 1846/11 the board considered the legal consequences that would ensue if there was no authorisation filed for a legal practitioner, or any subsequent approval by the appellant submitted for steps taken by a legal practitioner without such authorisation. Mr K. was a legal practitioner entitled to act as a representative under Art. 134(8) EPC. A legal practitioner must file the original version of a signed authorisation or a reference to a general authorisation already on file under R. 152(1) EPC and Art. 2, first sentence, of the Decision of the President of the EPO on the filing of authorisations (OJ SE 3/2007, 128).

In the case in hand Mr K. was informed that no authorisation was on file. In a reply to the board's invitation, Mr K. filed a duly signed general authorisation from the appellant dated 8 April 2016 (original and copy) within the two-month period set by the board. The board was satisfied that this general authorisation authorised Mr K. to represent the appellant in proceedings before the EPO and thus in the present appeal proceedings as from 8 April 2016. However, since the general authorisation was dated 8 April 2016, any procedural steps taken by Mr K. as representative up to that date were not covered by that authorisation itself.

The board referred to R. 152(11) EPC, which provides that the authorisation of an association of representatives is deemed to be an authorisation of any representative who can provide evidence that he/she practises within that association. However, a legal practitioner could not be treated as a member of an association of representatives within the meaning of R. 152(11) EPC and was therefore not covered by the legal fiction of that provision (see J 8/10, OJ 2012, 470).

In the specific circumstances of the case in issue, where a general authorisation dated 8 April 2016 was on file and the previous authorisation had allegedly been misplaced, the board would exceptionally have accepted that the appellant subsequently approve the procedural steps taken by Mr K. as representative in the period from 1 October 2010 to 8 April 2016. However, Mr K. failed to file any such approval. For the above reasons, the board could not accept that Mr K. had been authorised before 8 April 2016 by a previous authorisation. The legal consequence was that the procedural steps taken by Mr K. were deemed not to have been taken (R. 152(6) EPC applied mutatis mutandis). Consequently the notice of appeal was deemed not to have been filed and an appeal did not exist. In the absence of an appeal, there was no basis for payment of the appeal fee, which had therefore to be reimbursed.

IV. PROCEEDINGS BEFORE THE EPO

A. Examination procedure

(CLB, IV.B)

In T 2324/14 the examining division had found that the auxiliary request "prima facie [did] not overcome the objections under Art. 84 EPC and Art. 83 EPC", and thus it did not give its consent under R. 137(3) EPC to the auxiliary request. Under R. 137(3) EPC, the examining division has discretion to deny its consent to an auxiliary request. The board noted that like any other decision open to appeal, the discretionary decision to deny consent to an amendment must be reasoned (R. 111(2) EPC). In giving reasons for its decision, the examining division had acted correctly in this respect. The board noted that the EPC does not define what it means for an examining division to give or deny its consent to an amendment under R. 137(3) EPC or how it should exercise its discretion under R. 137(3) EPC. The boards of appeal had, however, accepted that the examining division could base its decision to deny consent to an amendment on prima facie considerations and that it could deny its consent to an amendment with prima facie deficiencies. A prima facie judgment was one which was made at first sight and was commonly understood to be one assumed to be correct until proven otherwise. Therefore, in making a prima facie judgment, the deciding body conceded that the judgment might not survive further scrutiny and might turn out to be incorrect on further investigation.

However, contrary to the wording of the decision, the examining division had not limited its examination of the auxiliary request to prima facie considerations. The board concluded that the discretionary decision was incorrect even though it was based on correct principles (see G 7/93, point 2.6 of the Reasons, OJ 1994, 775). In fact, the examining division had considered the auxiliary request fully, since it was able to give sufficient reasons for its conclusion that a patent could not be granted on the basis of that request. Since the examining division had considered the auxiliary request fully, Art. 12(4) RPBA obliged the board to take the first auxiliary request into account.

B. Special features of opposition and appeal proceedings

1. Apportionment of costs

1.1 Acts or omissions prejudicing the timely and efficient conduct of oral proceedings

(CLB, IV.C.6.2.2)

In T 2377/13 the respondent (opponent) attended the oral proceedings before the board, having learned that the appellant would not be there just two days beforehand because it had informed only the board of its absence. The respondent requested a different apportionment of costs, citing T 937/04 in which the board had held that, where a party decided very late on – in that instance a mere few days beforehand – not to attend the oral proceedings, it ran the risk of having costs awarded against it to compensate the other parties for any unnecessary expense they had incurred in attending. Such a decision, the board had held, was at its discretion and justified where the party had not shown due care, i.e. in cases of culpable actions of an irresponsible or even malicious nature. In T 2377/13, by contrast, the board held that the standard to apply in exercising its discretion on whether to apportion costs differently was that laid down in T 383/13 and T 1079/07. In the latter the board had diverged from T 937/04, holding that all parties to proceedings before the EPO had the right to be heard at oral proceedings to which they had been summoned, but were not obliged to attend. Informing the EPO and any other party to the proceedings of an intended absence in good time before the date scheduled was thus a matter of courtesy and respect, not a procedural obligation. In the case now in hand, the board would certainly not have cancelled the oral proceedings, even if informed well in advance of the appellant's absence, because it had wanted to be able to decide on and close the case on the date scheduled for them, and so they had definitely not been superfluous. Moreover, if the respondent had no longer wished to attend, it could have cancelled its hotel bookings as soon as it had learned of the appellant's absence, thus avoiding the costs incurred.

In T 1699/15, the appellant (patent proprietor) gave notice that it would not attend the oral proceedings only at a late stage: the day before them, at 2 p.m. The board decided not to apportion costs differently, in particular because there was no objective evidence to suggest deliberately ambiguous or abusive conduct. The parties had been summoned to the oral proceedings not only because of the auxiliary request of the respondent (opponent) but also in view of the need to discuss, among other matters, the question of a technical prejudice, as indicated in the board's preliminary opinion. In the absence of any further information from the board to say that the oral proceedings might be cancelled following the appellant's notice of absence, the respondent had had no guarantee that the decision would be in its favour and so had had to prepare to defend its case.

In T 258/13 the appellant withdrew its request for oral proceedings two days before the oral proceedings, which de facto amounted to only one day in view of the lateness in the day (5 p.m.). This was not notification in good time. Citing T 556/96, the board held that the appellant's request had to be treated as having been received so late that the respondent would already have had to be fully prepared, taking further into account the necessary travelling time during the day before the oral proceedings. An apportionment of costs in favour of the respondent was appropriate. However, for the oral proceedings only the presence of an authorised representative was necessary. The attendance or not of an accompanying person had no bearing on the conduct of the oral proceedings and was a matter of a deliberate choice by a party in which the other party did not need to be involved. Also to charge the appellant costs incurred by the accompanying person would contravene the principle of equity.

C. Opposition procedure

1. Amendments in opposition proceedings

(CLB, IV.D.4)

In T 2290/12, the appellant had used parts of a claim found by the opposition division to lack inventive step in a number of new independent claims which also included features taken from the description. The board considered such an approach to be legitimate, provided it did not amount to an abuse of procedure and the number of independent claims was not unreasonably high. Since lack of inventive of step was a ground for opposition, R. 80 EPC had in any event not been infringed.

In the proceedings underlying T 1758/15 the opposition division had concluded that four attempts to overcome a single issue were enough. The decision not to admit a further request was taken without having identified any signs of procedural abuse and without knowing the content of the further request. The board concluded that the opposition division had not exercised its discretion under R. 116(2) and Art. 114(2) EPC in a reasonable way, which constituted a substantial procedural violation.

2. Correction of the name of the opponent

(CLB, IV.D.2.2.4)

In T 579/16, the board confirmed that an opponent's name could be corrected under R. 139, first sentence, EPC where the conditions set out in G 1/12 (OJ 2014, A114) were met. Drawing on the wording of Section 121(1) of the German Civil Code, it interpreted the condition that the correction request be filed "without delay" to mean that it had to be filed "without culpable delay", i.e. how promptly a requester could reasonably be expected to act had to be assessed subjectively rather than objectively, having regard also to any special circumstances of the particular case in hand (J 16/08).

3. No consideration of late-filed evidence of public prior use

(CLB, IV.C.1.3.17 a))

In T 1955/13 the opposition division had held the opponent's late-filed allegation of its own public prior use to be an abuse of procedure, and so disregarded it, invoking decisions which, however, had both been taken in cases where a party had been accused of having deliberately withheld prior art that came from its own company in order to gain an advantage in the proceedings.

The board decided that the alleged prior use was not to be admitted into the proceedings because unsubstantiated. It went on to discuss in detail the inadmissibility of the prior use due to abuse of procedure, albeit without deciding the point.

The board considered that there was no indication in the case in hand that the opponent had deliberately withheld information already known to it. Therefore the question was whether omitting to conduct an in-house prior-art search at the start of the opposition proceedings only to submit, at a later stage (in this case shortly before the oral proceedings before the opposition division), purportedly novelty-destroying prior art found in house ultimately had to be treated as amounting to an abuse of procedure when it came to deciding on the submission's admissibility.

The board concluded that an opponent conducting business in the same technical field as the patent proprietor, and who therefore had reason to presume that it might find conflicting prior art in house, could rightly be expected to try to search for and file it as early as possible. That would mean conducting an appropriate search at the latest when preparing the opposition. Where such a search was omitted, be it wilfully or negligently, and nothing changed during the opposition proceedings that would prompt a reasonable party to search in a particular direction for the first time, it would be at odds with fairness and equal treatment to admit late-filed prior art found in house.

D. Appeal procedure

1. Filing and admissibility of appeal

1.1 Entitlement to appeal – party adversely affected

(CLB, IV.E.2.4.2 a))

Art. 107 EPC provides that any party to the proceedings adversely affected by a decision may appeal. For appellants to be adversely affected, the decision they are contesting must fall short of what they requested. In T 735/13, the board held that appellants who had agreed with a decision at the time but later changed their minds could not be regarded as adversely affected; an adverse effect required a discrepancy between the decision and what they had been requesting when it was issued. Opponents seeking to contest a patent grant had to set out why it was wrong. They could not be treated as contesting a patent's maintenance in a particular version if they did not make a case against that version based on grounds prejudicial to maintenance. It was only by citing one or more such grounds that they were able to show that they disagreed with the patent as granted or maintained. In the case in hand, the opponent had not, during the oral proceedings before the opposition division, challenged the auxiliary request at issue on the basis of any of the objections under Art. 100(a), (b) or (c) EPC now raised in its grounds of appeal, and its appeal was therefore rejected as inadmissible.

1.2 Entitlement to appeal – procedural status of the parties

(CLB, IV.E.2.4.3)

In T 540/13 the board stated that when an opposition division decided that an opposition was inadmissible in a case where at least one other admissible opposition had been filed, it was not necessary for the opponent whose opposition had been held inadmissible to appeal this decision in order to preserve its party status in appeal proceedings initiated by another party.

1.3 Form and content of notice of appeal (Rule 99(1) EPC)

(CLB, IV.E.2.5.2)

In T 2561/11 several issues arose with regard to the admissibility of the appeal. The notice of appeal had only been signed by the representative without mentioning the opponent. The board noted that neither the professional representative nor his law firm had been parties to the opposition proceedings. However, the representative had acted for the opponent before the opposition division. The board was satisfied that a reader willing to understand would have understood, before the end of the time limit for filing an appeal, that the notice of appeal had been filed on behalf of the opponent. There was no reasonable doubt as to the identity of the appellant. The board referred to G 1/12 (OJ 2014, A114), where it was established that an erroneous indication of the name and address of the appellant in the notice of appeal could be corrected under R. 101(2) EPC.

A further issue in T 2561/11 was that the notice of appeal only stated that the appellant was filing an appeal "against the decision of the opposition division". The board summarised the established jurisprudence with regard to the meaning of the expression "a request defining the subject of appeal" within the meaning of R. 99(1)(c) EPC and stated that the boards had repeatedly construed the appeal of patent proprietors against a decision to revoke the patent as a request that the decision be set aside in its entirety, even though there had been auxiliary requests before the opposition division (see T 358/08). Similarly, in appeals against the rejection of an opposition, the appeal of the opponent was construed as a request to set aside the decision under appeal and revoke the patent (see T 9/08, T 183/12 and T 256/13).

1.4 Statement of grounds of appeal

(CLB, IV.E.2.6.)

Under Art. 108 and R. 99(2) EPC, for an appeal to be admissible, the appellant must set out the reasons for setting aside the decision impugned, or the extent to which it is to be amended, and the facts and evidence on which the appeal is based. In T 393/15, the appellant had based its statement of grounds of appeal solely on an objection under Art. 83 EPC it had not raised at first instance. During the proceedings before the opposition division, however, features from the description had been added to independent claim 1 of the request ultimately held allowable. The board observed that a request amending a patent by adding features not in the claims as granted was as a rule open in opposition proceedings to objection under Art. 83 EPC (in conjunction with Art. 101(3) EPC), even if the ground for opposition under Art. 100(b) EPC had not been cited (see also T 66/14). Accordingly, the Art. 83 EPC objection raised on appeal did not go beyond the legal scope of the case before the opposition division, and the appeal was held admissible.

2. Interlocutory revision

(CLB, IV.E.2.9.1)

In T 2445/11 the application was refused for not meeting the requirement of R. 43(2) EPC. With the statement of grounds of appeal, the appellant filed a new set of claims which overcame the objections. The board accepted in principle that there are cases where amended application documents presented on appeal as the main request overcome the grounds for refusal yet do not warrant rectification of the decision because of other obvious deficiencies: deficiencies which are newly introduced and immediately apparent, or deficiencies which are well explained by remarks or objections included as obiter dicta in the decision to refuse and on which the applicant has had an opportunity to comment. In this respect, the board took the view that the expression "considers the appeal to be ... well founded" in Art. 109(1) EPC left the examining division room to exercise its judgement while bearing in mind that the purpose of interlocutory revision was to speed up the procedure. But where an application had been refused on grounds that were subsequently overcome, and more fundamental objections had not been well developed in the proceedings up to the refusal, there was little point in remitting the case to the boards of appeal.

3. Subject-matter under examination and non-opposed dependent claims

(CLB, IV.E.3.2.)

In T 2094/12 granted claim 5, which was dependent on granted claim 1, was not opposed. Claim 1 as upheld by the opposition division combined granted claim 1 with the (non-optional) subject-matter of granted claim 5 and was now the main request. The subject-matter of claim 1 of the main request was therefore based on "dependent subject-matters" as referred to in G 9/91 (OJ 1993, 408), according to which: "even if the opposition is explicitly directed only to the subject-matter of an independent claim of a European patent, subject-matters covered by claims which are depending on such an independent claim may also be examined as to patentability, if the independent claim falls in opposition or appeal proceedings, provided their validity is prima facie in doubt on the basis of already available information (cf. T 293/88, OJ 1992, 220). Such dependent subject-matters have to be considered as being implicitly covered by the statement under R. 55(c) EPC 1973 (now R. 76(2)(c) EPC)". Thus, in the case in hand, because the opposition notice was explicitly directed to granted claim 1, it was implicitly also directed to the subject-matter of granted claim 5, now claim 1 of the main request. The opposition division was thus also competent to consider the prima facie validity of the patent. In the case in hand the opposition division, applying point 11 of the Reasons of G 9/91, had held that the validity of the claims as upheld was not prima facie called into doubt by the evidence (documents) then on file. Therefore, in order to challenge this decision on its merits, the appellant (opponent) had to demonstrate why the division had been wrong in this finding of prima facie validity and should have carried out a full examination. The review of the decision had to take place under the same constraint, i.e. it had to consider prima facie validity first, and only if that was not confirmed could a full examination take place. The board concluded that the arguments presented by the appellant (opponent) had failed to cast doubt on the prima facie validity of the claims and dismissed the appeal.

4. Review of first-instance decisions

(CLB, IV.E.3.6)

4.1 The boards' discretion

4.1.1 Scope for reviewing discretion

In several decisions, the boards have addressed the issue of their scope for reviewing decisions taken by departments of first instance in exercise of their discretion. Among the factors to be considered is whether or not the department of first instance exercised its discretion properly.

It is established case law that, on appeal against a decision taken by a department of first instance in exercise of its discretion, it is not for the board to review all the facts and circumstances of the case as if it were in that department's place and decide whether or not it would have exercised discretion in the same way. If the EPC requires that a department of first instance exercise discretion in certain circumstances, that department should have a certain degree of freedom to do so without interference from the boards. They should overrule the way in which it exercised its discretion in reaching a decision in a particular case only if they conclude that it did so in accordance with the wrong principles, without taking the right principles into account or in an arbitrary or unreasonable way, thereby exceeding the proper limits of its discretion (on this point, see, in particular, G 7/93, OJ 1994, 775).

Although G 7/93 was concerned with a specific situation, namely an examining division's refusal to admit amendments after issuing a communication under R. 51(6) EPC 1973, the boards have applied the criteria established there to their review of other discretionary decisions of the departments of first instance (T 820/14).

In T 820/14 the board had to decide whether it could and should admit an auxiliary request not admitted by the examining division but resubmitted on appeal. Art. 12(4) RPBA gave it the power to hold inadmissible submissions not admitted at first instance, but that could not be interpreted as meaning it was obliged to do so. So long as departments of first instance had exercised their discretion properly, the boards as a rule should not overrule their decisions and substitute their own discretion for that exercised at first instance. That applied regardless of whether not they ultimately decided to admit the previously refused submissions, uphold the refusal to admit them on different grounds or remit the case to the department of first instance and give it an opportunity to exercise its discretion anew. By the same token, however, the case law established in G 7/93 (OJ 1994, 775) could not be interpreted as imposing on the boards any duty to uphold a proper exercise of discretion at first instance. In the case in hand, the board exercised its own discretion and admitted the second auxiliary request.

In T 1816/11, the examining division had refused to admit the main request in exercise of its discretion under R. 137(3) EPC. The appellant resubmitted the request with its statement of grounds of appeal and asked the board to admit it. The board found that the power under Art. 12(4) RPBA not to admit requests implied a matching power to admit them, even if, according to the established case law, the boards' competence to review exercises of discretion at first instance was limited in certain circumstances. Summarising G 7/93 (OJ 1994, 775), the board observed that it had been concerned with a discretionary decision of an examining division relating only to the conduct in the proceedings before it, the main point being the lateness of an amendment. The case now in hand was different in that the reason for the examining division's refusal to admit the main request had not been its late filing but solely that it did not meet the requirements of Art. 84 and 56 EPC. Since the discretionary decision not to admit the request had thus been taken for substantive reasons only, and not for procedural reasons, the criteria established in G 7/93 could not apply. Rather, what the board had to review was the substantive assessment (clarity, inventive step) underlying the discretionary decision, which went to the very essence of the boards' power of review. The board thus admitted the main request on appeal.

4.1.2 Own discretion to admit submissions not admitted at first instance

In a number of decisions, the boards have observed that, under Art. 12(4) RPBA, they generally have their own margin of discretion to admit on appeal facts, evidence and requests refused by a department of first instance in proper exercise of its discretion.

In T 945/12 the board referred to some decisions where it was stated that in the case of a review of first-instance discretionary decisions a board of appeal nevertheless had to exercise its discretion under Art. 12(4) RPBA independently, giving due consideration to the appellant's additional submissions. In doing so, the board of appeal was not re-exercising the discretion of the department of first instance based on the case as it was presented then, but rather taking into account additional facts and different circumstances while exercising its own discretion under Art. 12(4) RPBA (see also T 2219/10, T 971/11).

In T 556/13 the board was aware of the line of jurisprudence of the boards of appeal establishing that, where it has to be decided under Art. 12(4) RPBA whether to hold a request not admitted into the first-instance opposition proceedings inadmissible in appeal proceedings, this usually amounts to a review of the opposition division's exercise of its discretion under R. 116 EPC and that, therefore, admitting such requests into appeal proceedings under Art. 12(4) RPBA undermines the discretion conferred on the opposition division, unless the opposition division did not properly exercise its discretion (see e.g. T 28/10). In the board's view, the jurisprudence is not to be understood as meaning that the discretion conferred on the boards of appeal under Art. 12(4) RPBA is limited to such an extent that the board must generally hold inadmissible a request which had not been admitted into the proceedings by the opposition division by a correct discretionary decision. Thus, it is not ruled out that, in view of the particular facts and circumstances of a particular case, the board will not hold a request inadmissible, although it was rightly not admitted into first-instance proceedings by the opposition division. This may occur, for example, if the board is confronted with additional facts and different circumstances or with additional submissions made by a party in the appeal proceedings. Another example could be that the reasons for the contested decision themselves contain considerations which go beyond those which were relevant for the opposition division's discretionary decision not to admit a request.

4.2 Review of an opposition division's discretionary decision (not) to admit a new ground for opposition

In T 1286/14 the board addressed the limited scope for reviewing exercises of discretion to refuse a "fresh ground for opposition" where the proprietor objects to its admission on appeal. In the case at issue, the opposition division had considered a late-filed ground irrelevant and refused to admit it. Since the proprietor (respondent) had expressly refused to consent to its admission on appeal, the board could not admit it. In such circumstances, it was sufficient for the board to establish that there was evidence that the opposition division had actually examined whether the ground was prima facie relevant and given reasons for its finding on this. In other words, the board's duty to review the exercise of discretion was limited. Citing G 10/91 (OJ 1993, 420), it thus refrained from examining the substance of the fresh ground, thereby following the approach taken in T 736/95, OJ 2001, 191; T 1519/08 and T 1592/09 and rejecting that entailing a thorough review of the earlier exercise of discretion on its merits, as taken in e.g. T 1053/05, T 1142/09 and T 620/08.

4.3 Review of discretion – stay of proceedings

In J 1/16, the applicant had appealed against the Legal Division's decision not to resume stayed grant proceedings until further notice. The Legal Board stated in its catchword: where, when deciding not to resume proceedings, the Legal Division has properly identified and exhausted its scope for discretion and weighed up the situation in the light of all the relevant factors, without being influenced by anything irrelevant and without making any logical mistakes in its assessment of the facts, it is not open to the Legal Board to exercise its own discretion in the Legal Division's place. The Legal Board therefore formally upheld the Legal Division's decision but varied its substance in view of how the circumstances had changed in the meantime (appeal in national entitlement proceedings had since been dismissed).

5. Late submission

5.1 Late-filed documents

(CLB, IV.C.1.3.14)

In T 2193/14 the board considered the admissibility of documents filed with the statement of grounds of appeal for the purpose of questioning the validity of the subject-matter of granted dependent claims against which no substantiated attack had been made in the notice of opposition. The board decided to exercise its discretion pursuant to Art. 12(4) RPBA not to admit into the appeal proceedings documents A6 to A10, submitted with the statement of grounds of appeal. These documents should and could already have been submitted to the opposition division. Considering that it was already questionable whether admitting A6 to A10 during the opposition proceedings would have been compatible with the legal and factual framework within which the substantive examination of the opposition in principle should be conducted (see G 9/91, OJ 1993, 408), and in the absence of any arguments in the notice of opposition, it was doubtful whether the validity of the dependent claims was indeed questioned simply by ticking the box in EPO Form 2300 that the patent was opposed as a whole.

5.2 Unsubstantiated requests

(CLB, IV.E.4.2.4)

According to Art. 12(2) RPBA the appellant must set out in writing the reasons as to why the decision under appeal should be "reversed, amended or upheld" and specify expressly all the facts, arguments and evidence relied on. This is a prerequisite in order for the matter to be taken into account (cf. Art. 12(4) RPBA). In T 2598/12 it was stated that logically there is no time bar to the requirement following from Art. 12(2) and (4) RPBA that a request filed during appeal proceedings must be properly substantiated. Consequently, this requirement applies, mutatis mutandis, to new requests filed in response to a communication of the board.

5.3 Late-filed requests

5.3.1 Response to new objections

(CLB, IV.E.4.4.3)

The board in T 2227/12 stated in its catchword: where only objections under Art. 123(2) and 84 EPC have been addressed in the contested decision and in the appeal proceedings, the board, in deciding whether to admit a request filed at the oral proceedings, need examine only whether it is prima facie allowable in view of those objections and need not examine its prima facie allowability under Art. 52(1), 54 and 56 EPC. The appellant in the case in hand had filed an auxiliary request at the oral proceedings with a view to overcoming an objection raised there for the first time. The board admitted the request. According to the boards' case law, amended requests filed in response to new objections were generally to be admitted if – as here – a party, no matter how diligent its procedural conduct, would have had no reason to make the amendment earlier. The contested decision had merely addressed objections under Art. 123(2) and 84 EPC although lack of novelty and lack of inventive step had also been raised as grounds for opposition. The board had therefore pointed out in an earlier communication that, if it were to find that the case turned on those aspects, it would remit the case to the opposition division.

5.3.2 Diverging versions of claims

(CLB, IV.E.4.4.4)

In T 1903/13 the board stated in its catchword that it is within the board's discretion not to admit auxiliary requests which define subject-matter "diverging" from subject-matter of higher-ranking requests, including those requests which, in essence, were filed during the first-instance proceedings and re-filed with the respondent's reply, but were not examined by the first-instance department.

5.3.3 Reverting to the patent as granted

(CLB, IV.E.4.5.1)

In T 1578/13, the appellant requested that its patent revoked by the opposition division be maintained on the basis of three auxiliary requests already dealt with in the contested decision. In a fourth auxiliary request, it requested that it be maintained as originally granted. In its first three auxiliary requests, the wording of the granted patent had been amended to read "gas outlet casing" instead of "gas inlet casing", which, it argued, corrected an obvious mistake in the granted version (R. 139, 140 EPC). The board first of all observed that G 1/10 (OJ 2013, 194) had ruled that R. 140 EPC could not be taken as a basis for correcting the wording of a patent, before finding that the auxiliary requests were anyway not allowable, because the correction they made was inadmissible in the absence of any obvious mistake (R. 139 EPC), while the resulting amendment would inadmissibly extend the scope of protection (Art. 123(3) EPC).

The fourth auxiliary request, which the appellant had filed only on appeal, contained the claims as granted. During the opposition proceedings, the appellant, instead of requesting that the opposition be rejected or defending the patent as granted, had merely requested that the patent be maintained in versions correcting the allegedly obvious mistake. The board considered that, at the latest on learning at the oral proceedings at first instance, that the opposition division was not minded to allow the requested correction, the appellant ought to have begun defending the patent as granted. By that point, it had to have been clear to it that reverting to the patent as granted would immediately overcome the objection under Art. 123(3) EPC. Its own conduct had thus prevented examination of the granted claims at first instance, so to allow it now to revert those claims on appeal would be incompatible with due procedural economy.

6. Reimbursement of appeal fees

6.1 Distinction between an appeal deemed not to have been filed and an inadmissible appeal

(CLB, IV.E.8.2.2)

In ex parte case T 2406/16, the appellant received a communication from the board registrar indicating that the fee was missing, and its request for re-establishment of rights was eventually refused. The board conceded that the appeal fee had been paid together with the request for re-establishment and the wording of Art. 108, second sentence, EPC would suggest that the appeal was deemed to have been filed on that day, but, having analysed the case law, came to the conclusion that it now seemed to be settled case law of the boards of appeal that an appeal is deemed not to have been filed where the appeal fee is paid after the two-month time limit of Art. 108, first sentence, has expired. As a consequence, the appeal was deemed not to have been filed, so the appeal fee had been unduly paid and was to be reimbursed.

6.2 Substantial procedural violation

6.2.1 Oral proceedings

(CLB, IV.E.8.4.2)

In T 679/14 the board held that it was contrary to the principles of procedural efficiency and legal certainty to repeatedly adjourn oral proceedings. Regarding postponements at the instigation of the division, the board quoted the Guidelines, E-II, 7.1, and concluded that the postponement of oral proceedings on five occasions at the instigation of the examining division without, as far as apparent from the file, serious reasons constituted a procedural deficiency in the circumstances of the case in issue (eight postponements altogether). The board nevertheless refrained from deciding whether it amounted to a substantial procedural violation. The appeal fee was reimbursed due to the violation of R. 111(2) EPC (insufficient reasons).

6.2.2 No reasons for refusal of request for postponement

(CLB, IV.E.8.4.2 d))

In T 1750/14 the appellant's representatives requested a postponement of the final date (under R. 116(1) EPC) repeatedly and separately from their request to postpone the date for oral proceedings. The specific reasons for not allowing any postponement of the final date ‒ regardless of the refusal of the request for postponement of the date for oral proceedings ‒ were not addressed in the decision of the examining division. It was only mentioned that the question of filing amendments of the application after expiry of the final date did not arise. This remark could not, however, be considered to be a reasoning for the refusal of the request for postponement of the final date. The fact that no submissions other than procedural requests were filed after the final date did not mean that the request for postponement of the final date became pointless. The request for postponement of the final date had never been withdrawn and the appellant's desire to file amended claims had been made clear even during oral proceedings before the examining division when it requested that the examination proceedings be continued in writing.

The board could only surmise that the examining division might have assumed that the final date was only to be postponed together with the date for the oral proceedings and that, as a consequence, the request for postponement of the final date and the request for postponement of the date for oral proceedings could not be separated from each other and could only be decided together. However, in this case, or indeed if the examining division considered those two requests to be linked in any other way, the appellant's arguments concerning the request for postponement of the final date should have been considered at least in the context of the request for postponement of the date for oral proceedings.

Therefore, the examining division had committed a substantial procedural violation.

Nevertheless the board held that the applicant ‒ and in particular its professional representative ‒ must or should have known, in view of the last sentence of R. 116(1) EPC that it is generally not guaranteed that any written submission is automatically admitted into the proceedings before the EPO for the sole reason that it is filed prior to the final date, nor is it entirely unlikely that a submission may be admitted at the department's discretion when filed after that date. The applicant could have followed the invitation from the examining division and attempted to file, for example, amended sets of claims with the aim of overcoming the objections raised in the summons. This, however, was not done by the applicant of its own volition. In the board's view, such procedural behaviour spoke against regarding the reimbursement of the appeal fee as equitable within the meaning of R. 103(1)(a) EPC. A party cannot gain a procedural advantage from an omission of its own.

E. Proceedings before the Enlarged Board of Appeal

1. Petition for review under Article 112a EPC

(CLB, IV.F.3)

In R 2/14 and R 3/15 the Enlarged Board allowed the petitions.

In the inter partes proceedings under review in R 2/14, the Enlarged Board noted in its decision of 22 April 2016 that the board's decisive line of argument had concerned the aspect of modifying the inactive SEQ ID NO: 4 by means of recloning the desaturase, starting from E. gracilis. The reasons given by the board were limited in so far as, after establishing the need for recloning, it had immediately stated its conclusion that, although the skilled person could in fact perform each of the necessary steps, combining those steps created an undue burden for him. The other two alternative approaches relied upon by the petitioner had not been discussed at all by the board; they had merely been referred to as suffering from the same negative conclusion. The board had mentioned neither facts nor a sequence of arguments that had led it to this conclusion. Therefore, the affected party could not understand the conclusion or how the board had arrived at it. The Enlarged Board allowed the petition.

In R 3/15 the Enlarged Board set aside decision T 1225/13 on account of a breach of the right to be heard. The board had reformulated the problem based on a new interpretation of claim 1, presented for the first time in the written reasons for its decision, that none of the parties had previously made a case for, either in the opposition or the appeal proceedings. It was apparent from the parties' submissions that not only the appellant but also the respondents had assumed a different interpretation of claim 1 in the discussion on feature (ii). Thus the appellant had been surprised by the board's new line of argument on inventive step in its written decision, on which it had not had an opportunity to comment. That was a breach of the right to be heard (Art. 113 EPC).

In R 2/15 the Enlarged Board held that petitions for review of interlocutory decisions were not generally inadmissible (see also R 5/08). The Enlarged Board saw no reason to assume that Art. 106(2) EPC, which stipulates that a decision which does not terminate proceedings as regards one of the parties can, as a rule only be appealed together with the final decision, was to be applied to the petition for review procedure. Neither Art. 112a EPC nor R. 104 to 110 EPC contained a provision corresponding to Art. 106(2) EPC.

In R 8/15 the Enlarged Board held that Art. 113(1) EPC must be interpreted more narrowly than R. 102(g) EPC. The latter requires a board to give reasons for its decision, but infringement thereof is not as such a ground for review. Reasons may be incomplete, but as long as they permit the conclusion that the board, in the course of the appeal proceedings, substantively assessed a certain point that it found to be relevant, there is no violation of Art. 113(1) EPC.

In R 6/16 the Enlarged Board found the (proprietor) appellant's petition for review unallowable. The appellant alleged that fundamental violations of its right to be heard (Art. 113(1) EPC) had occurred in the appeal proceedings on the basis that the written reasons for the challenged decision revealed that the board had made several erroneous assumptions which had not been laid open to it in the oral proceedings. The Enlarged Board addressed the appellant's numerous complaints, reiterating that the right to be heard does not impose a legal obligation on a board to disclose in advance to the parties how and why it would come to its conclusion on the basis of the decisive issues under discussion. This is part of the reasoning given in the written decision (R 1/08, R 15/12, R 16/13). Grounds or evidence within the meaning of Art. 113(1) EPC need not emanate from the board; it is sufficient if another party raises the objection (R 2/08). The Enlarged Board thus concluded there had been no fundamental violation of the appellant's right to be heard where it itself submitted the board was merely adopting the arguments of the opponent. A violation of Art. 113 EPC can also only be considered fundamental within the meaning of Art. 112a(2)(c) EPC if there is a causal link between the alleged violation and the final decision (R 1/08, R 11/09, R 19/09). In this case, there was not.

V. DISCIPLINARY BOARD OF APPEAL

1. Objective review of the marks awarded

(CLB, V.2.6.3)

One of the arguments advanced by the appellant in EQE case D 20/16 was that Paper A, like the pre-examination, comprised a fixed set of "questions" for which marks were awarded according to a strict scheme which did not allow any discretion, and that accordingly the independent claim could be marked using a checklist and an Excel spreadsheet listing desired features, equivalents, and total points for given combinations. The Disciplinary Board of Appeal could assess the correct marks without an in-depth review or interfering with the value judgment of the Examination Board.

The Disciplinary Board did not agree. In the pre-examination (R. 10 IPREE), candidates were expected to respond to clearly defined questions or statements in a multiple choice mode, to which they could only answer "true" or "false" by ticking a box; they had no possibility to add any reasons or explanatory notes. That was the decisive difference to the EQE, in particular Paper A under R. 23 IPREE, where the candidate could offer solutions which he found met the requirements of R. 23 IPREE, and even, if need be, give reasons for his proposals in supplementary notes (see R. 23(6) IPREE). The task set was a request to draft a set of claims and an introductory part of the description for a European patent application which would protect the client's invention, leaving considerable scope for individual decisions about the draft required. It was therefore not comprehensible how and to what extent "the mistake" could have been, as the appellant contended, "the result of an inconsistently or incomprehensibly formulated question/task and should be deemed obvious in accordance with D 13/02". The appellant's submission had been taken almost literally from decision D 3/14, which, however, pertained to the pre-examination and was not significant for the case in hand.

The members of the Examination Board have a broad discretion when awarding marks based on their assessment of the candidate's solution. It was not possible for the Disciplinary Board to assess whether, for example, the inclusion or exclusion of feature d2b in the independent claim was an example of an inconsistently or incomprehensibly formulated question/task, without conducting an in-depth review of the entire content of the examination or interfering with value judgements of the Examination Board.

VI. THE EPO ACTING AS A PCT AUTHORITY – EURO-PCT APPLICATIONS

1. Competence of the EPO acting as designated or elected Office

(CLB, VI.3)

In J 19/16 the Legal Board came to the conclusion that the loss of rights, i.e. the deemed withdrawal of the application, which had occurred in the international phase could no longer be remedied in the national phase.

The Legal Board pointed out that, under Art. 24(1)(ii) PCT, possible deficiencies in the international application occurring during the international phase were generally relevant for the application's effect as a direct European application and as a rule came to bear in respect of the latter. Furthermore, the competence of the designated Office to process or examine the application (only) after effective entry into the national phase (Art. 23 and 22 PCT) did not prevent the receiving Office from taking a procedural step regarding the non-payment of fees prescribed under Art. 3(4)(iv) and R. 27.1 PCT in the international phase. Nor did the PCT contain a provision – complementary to the provision of Art. 23 PCT – which generally denied the competence of the receiving Office to take procedural steps once the application had entered the national phase. Hence, parallel competences of the receiving Office and the designated Office in respect of the same application were not excluded.

The Legal Board stated that, as a further exception to the principle underlying Art. 24(1)(ii) PCT, the EPO acting as designated Office by virtue of Art. 24(2) PCT could maintain the effect of the international application as a European patent application pursuant to Art. 11(3) PCT for other reasons, if the loss of rights occurred due to a mistake made by the applicant. The two-month time limit pursuant to Art. 25 PCT was not to be applied to circumstances that were to be considered under Art. 24(2) PCT. Thus, the Legal Board decided that the time limit, if any, for making a request to be excused under Art. 24(2) PCT was subject to national law, and to national law only.

 

 

[ *** ] Link available as soon as possible in the HTML version.

267 references found. Limiting display to 200 references.

Click X to load a reference inside the current page, click on the title to open in a new page.

EPC Articles

EPC Implementing Rules

PCT Articles

PCT Implementing Rules

Case Law of the Enlarged Board

General Case Law